Vous êtes sur la page 1sur 45

Robert Krzyzanowski Solutions to Dummit and Foote Chapter 1

Chapter 1.1
These problems implicitly make use of the following lemma (stated casually in the book).

Lemma If aK b is a group and L K is closed under , then is associative in L .

closed, + a, - b a+ ,b - L . However, since + , - L implies + , - K we also know


Proof. We know is associative in K by definition of a group. Now let + , - L . Since L is

+ a, - b a+ ,b - . Hence, is associative in L .

Problem 1.1.1 Determine which of the following binary operations are associative:
(a) the operation on defined + , + ,
(b) the operation on defined by + , + , +,
+,
(c) the operation on defind by + ,
(d) the operation on defined by a+ ,b a- . b a+. ,- ,.b
&

(e) the operation on e!f defined by a b +, .

Solution. (a) Not associative. For example, a# "b " ! # # a" "b.
(b) Associative, because
a+ ,b - a+ , +,b - a+ , +,b - a+ , +,b-
+ a, - ,- b +a, - ,- b + a, - b +a, - b + a, - b.
The intermediate steps follow because usual addition and multiplication is associative and commutative in .
(c) Not associative. For example, a! !b #& & " ! a! #&b.
(d) Associative, because
aa+ ,b a- . bb a/ 0 b a+. ,- ,.b a/ 0 b aa+. ,- b0 a,.b/ a,.b0 b
a+a.0 b ,a-0 ./b ,a.0 bb a+ ,b a-0 ./ .0 b
a+ ,b aa- . b a/ 0 bb.
Notice we could not say a b is isomorphic to a b even though intuitively +
, -
. +,,-
,. ,
because this would exclude , or . equal to ! (which is encompassed by the former).
(e) Not associative. For example, a" "b # "
# # " a" #b.

Problem 1.1.2 Decide which of the binary operations in the preceding exercise are commutative.

Solution. (a) Not commutative. For example, 1 ! " " ! ".


(b) Commutative, because
+ , + , +, , + ,+ , +,
due to addition and multiplication being commutative in .
(c) Commutative, because
+, ,+
+ , & & , +,
due to addition being commutative in .
Robert Krzyzanowski Solutions to Dummit and Foote Chapter 1

(d) Commutative, because


a+ ,b a- . b a+. ,- ,.b a-, .+ .,b a- .b a+ ,b,
due to addition and multiplicaiton being commutative in .
"
(e) Not commutative. For example, # " # # " # .

Problem 1.1.3 Prove that the addition of residue classes in 8 is associative (you may assume it is well
defined).

Proof. Let + , - 8. Then + , - + , - (by definition--see page 9 in the book),


which equals + a, - b + , - (again by definition). However,
+ , - a+ , b - + , - + , - . 

Problem 1.1.4 Prove that the multiplication of residue classes in 8 is associative (you may assume it is
well defined).

Proof. Let + , - 8. Then +, - + ,- (by definition--see page 9 in the book), which


equals +a,- b +,- (again by definition). However,
+,- a+,b- +, - + ,- . 

Problem 1.1.5 Prove for all 8 " that 8 is not a group under multiplication of residue classes.

Proof. In the book, we've seen a8b is a group. Hence, ! must be the guilty element of breaking
this structure. Indeed, ! has no inverse, since ! + ! + ! + ! + ! for any + 8, and we
know " is the identity since " + " + + + " + " for any + 8. Since there is no element +
such that ! + ", ! has no inverse and by definition 8 is not a group under multiplication of residue
classes. 

Problem 1.1.6 Determine which of the following sets are groups under addition:
(a) the set of rational numbers (including ! !") in lowest terms whose denominators are odd
(b) the set of rational numbers (inculding ! !") in lowest terms whose denominators are even
(c) the set of rational numbers of absolute value "
(d) the set of rational numbers of absolute value " together with !
(e) the set of rational numbers with denominators equal to " or #
(f) the set of rational numbers with denominators equal to ", #, or $.

Solution. For each respective problem, call the group K.


(a) This is a group. First, if +, .- K with # l , and # l . (i.e., both are odd) and a+ ,b a- .b ", then
+ - +.,-
, . ,. K
since # l ,. ; hence we have closure. We know it is associative since is on addition and K . The
identity is !" since !" +, +, +, !" for all +, K. Finally, each element has an inverse since
+ + ! +
, , " for any , K .
"
(b) This is not a group because it does not have closure. For example, # K but "# "# "
" K since "" is in
lowest terms and the denominator is odd (not even).
Robert Krzyzanowski Solutions to Dummit and Foote Chapter 1
"
K since "# " but
# # " K since k"k
(c) This is not a group because it does not have closure. For example, #
" "
".
(d) This is not a group since it fails closure. For example, $# " K since $# k"k ", but $
a"b
" " and "# !.
#
"
# K since #

(e) Assume each rational number is in lowest form. This is a group. First, take +, .- K with the greatest
common divisor of + and ,, and - and . equal to 1. Consider , . #. Then +# #- +- # K . Otherwise,
+ - +.,-
, . ,. K since ,. " or # since gcda,. +. ,- b " or # (if ,. # and +. ,- is even, the
gcd becomes "). Hence, K is closed. We know it is associative since is on addition and K . The
identity is !" since !" +, +, +, !" for all +, K. Finally, each element has an inverse since
+ + ! +
, , " for any , K . By definition, K is a group.

(f) This is not a group because it is not closed. For example, "# "
$ K but
"
# "
$ "
' K. 

Problem 1.1.7 Let K eB l ! B "f and for B C K let B C be the fractional part 90 B C (i.e.,
B C B C cB Cd where c+d is the greatest integer less than or equal to +). Prove that is a well-
defined binary operation on K and that K is an abelian group under (called the real numbers mod ").

former case, cB Cd ! so that B C B C cB Cd B C. Otherwise, cB Cd " so that B C


Proof. To show the operation is well-defined, notice either ! B C 1 or " B C #. In the

B C ". Hence, is a well-defined binary operation on K. To show closure, again consider the two cases
mentioned earlier. In the former, B C B C and since ! B C B C 1 by assumption, B C K.
In the latter case, B C B C " and since " B C # we have " " ! B C " B C
# " " so again B C K. Therefore, K is closed. To show it is associative, notice for B C D K,
aB Cb D aB C cB Cdb D aB C cB Cdb D cB C cB Cd D d
B C D cC D d cB C D cC D dd B aC D b cB aC D bd B aC D b.
The middle equality holds because cB Cd cB C cB Cd D d cC D d cB C D cC D dd which

and " C D #. Then cB Cd cC D d " so the equation holds. Otherwise, assume without loss of
needs to be explicitly justified case-by-case. Assume ! B C " and ! C D ", or " B C #

generality ! B C " and " C D #. Then cB Cd ! and cC D d ", so that


cB Cd cB C cB Cd D d cB C Dd " cB C D "d cC Dd cB C D cC Ddd.

! B c! Bd B ! cB !d B cBd for any B K. Finally, each element has an inverse, since


Hence, the operation is associative. Furthermore, ! is the identity since

B aBb cB aBbd aBb B caBb Bd ! c!d ! for each B K. Therefore, K is a group.


Finally, K is abelian since for any B C K, we have that B C cB Cd C B cC Bd since addition
is commutative in . Hence, K is an abelian group. 

Problem 1.1.8 Let K eD l D 8 " for some 8 f.


(a) Prove that K is a group under multiplication (called the group of roots of unity of ).
(b) Prove that K is not a group under addition.

(a) To prove closure, let A D K. Then b8 7 such that A8 D 7 ". Then


aAD b aA b aD b "7 "8 " and since 87 (the positive integers are closed under
Proof.
87 8 7 7 8

multiplication), by definition AD K. Hence, K is closed. Associativity is guaranteed since e!f is a group


under multiplication, and K e!f (notice ! K since there is no 8 such that !8 "). The identity
Robert Krzyzanowski Solutions to Dummit and Foote Chapter 1

is " since for 8 " we have "" " so that " K, and furthermore for all D K, " D D " D .
Finally, each element has an inverse since for each D K there is an 8 such that D 8 ", so that
D 8" D D D 8" D 8 ". Therefore, aK b is a group. 
(b) Since " K, it can not be a group under multiplication since " " # K as there is no 8 such
that #8 " (and hence K is not closed). 

Problem 1.1.9 Let K + ,# l + , .


(a) Prove that K is a group under addition.
(b) Prove that the nonzero elements of K are a group under multiplication.

Proof. (a) Let + ,# - . # K. Then + ,# - .# a+ - b a, .b#


is in the group, because + - , . (since a b is a group). Associativity is guaranteed since K
and a b is a group. The identity is ! !# since ! !# + ,# + ,#
! !# + ,# for any + ,# K. Finally, each element has an inverse since for
+ ,# K, + ,# + a,b# + a,b# + ,# ! !#.
Therefore, K is a group under addition. 

(b) Let + ,# - . # K. Then + ,#- .# a+- #,.b a+. ,- b# is in the


group, because +- #,. +. ,- (since a b is a group). Associativity is guaranteed since K
and a b is a group. The identity is " !# since " !#+ ,# + ,#
" !# + ,# for any + ,# K. Finally, each element has an inverse since for
+ ,# K, + ,# +# #,
+
#
, # +# #,
+
#
, #+
,# " !# (this was
# K since
+# #,# +# #,#
+ ,
obtained by solving for - and . in +- #,. " +. ,- !). We know +# #, # +# #, #
+ ,

both the terms be ! since ! Ke!f). Therefore, Ke!f is a group under multiplication. 
+# #,# +# #,# K (notice the denominator can never be ! or that would contradict + , , and neither can

Problem 1.1.10 Prove that a finite group is abelian if and only if its group table is a symmetric matrix.

Proof. Assume K e1" 18 f is a finite abelian group. Then the 3, 4 entry in its group table is the

definition, the group table is a symmetric matrix. Now assume K e1" 18 f is a finite group with a
group element 13 14 14 13 . The 4 3 entry in its group table is the group element 14 13 13 14 . Hence, by

symmetric matrix. Then the 3, 4 entry is the same as the 4 3 entry, that is, 13 14 14 13 . However, this holds for
any two elements 13 14 K so that 13 14 14 13 for all elements of K. This is precisely the definition of an
abelian group. 

Problem 1.1.11 Find the orders of each element of the additive group "#.

Solution. The group is "# ! " # "". Then the orders, respectively, are ", "#, ', %, $, "#, #,
"#, $ % ' and "#. Notice these are kKkgcdaB kKkb. Indeed, this will be proven later. 

Problem 1.1.12 Find the orders of the following elements of the multiplicative group a"#b : " ", &, (
, (, "$.
Robert Krzyzanowski Solutions to Dummit and Foote Chapter 1

Solution. The identity is ", so the order for B is the smallest 8 e_f such that B8 " (with
B_ "). Respectively, these are " # # # %, and " (since "$ "). 

Problem 1.1.13 Find the orders of the following elements of the additive group $': " #, ', *, "! "#
" "! ").

Solution. The identity is !, so the order for B is the smallest 8 e_f such that 8B ! (with
_B !). Respectively, these are $' ") ' % ") $ $' ") #. 

Problem 1.1.14 Find the orders of the following elements of the multiplicative group a$'b : " " &
"$ "$ "(.

Solution. The identity is ", so the order for B is the smallest 8 e_f such that B8 " (with
B_ "). Respectively, these are " # ' $ ' #. 

Problem 1.1.15 Prove that a+" +# +8 b" +8" +8"


"
+""

Assume a+" +# +8 bB " so that B a+" +# +8 b" . Then +"" a+" +# +8 bB +"" " so that
a+"" +" ba+# +$ +8 bB a+# +$ +8 bB +"" . Similarly, a+$ +% +8 bB +#" +"" . Applying this 8 times results
Proof.

in B +8" +8"
"
+"" , as desired. 

Problem 1.1.16 Let B be an element of K. Prove that B# " if and only if kBk is either " or #.

Assume B# ". If B ", then kBk ". Otherwise, kBk " (only the identity has order ") so
that kBk # by definition since # would be the smallest power B need be raised to in order to obtain the
Proof.

identity. On the other hand, assume kBk is either " or #. If kBk ", then B " as only the identity has order ".
Otherwise kBk # so by definition of order, B# ". 

Problem 1.1.17 Let B be an element of K. Prove that if kBk 8 for some positive integer 8 then
B" B8" .

Proof. Let kBk 8. By definition, B8 ". Hence, B B8" B8" B ". This is precisely the
definition of B" B8" . 

Problem 1.1.18 Let B and C be elements of K. Prove that BC CB if and only if C" BC B if and only if
B" C" BC ".

Proof. Assume B C K and BC CB. Multiplying by C" on the left, C" BC C" CB B. Now
assume C BC B. Multiplying by B" on the left, B" C" BC B" B ". Finally, assume B" C " BC ".
"

Multipling by CB on the left, aCBbB" C" BC CaB B" bC" BC [generalized associativity] aC C" bBC
BC " aCBb CB. 

Problem 1.1.19 Let B K and let + , .


(a) Prove that B+, B+ B, and aB+ b, B+, .
(b) Prove that aB+ b" B+ .
(c) Establish part (a) for arbitrary integers + and , (positive, negative, or zero).
Robert Krzyzanowski Solutions to Dummit and Foote Chapter 1

Proof. Notice it is obvious B+ B+" B for all + . This is because we can recursively define B+ .
If + !, then B+ ". Otherwise, B+ B+" B. Similarly, B+ B +" B" .
(a) We will induct on + and , using strong induction. First, notice B"" B# B B [definition] B" B" .

Ba5"b7 B5" B7 for all 7 5 ". First, B5" B5 B" so that B5" B B5 BB B5 aBBb B5 B#
Now assume B87 B8 B7 for all 7 8 and 8 5 for some 5 . Then inductively we show

[definition] B5# . The last step follows because if 5 ", B" B# BBB B$ B"# . Otherwise, we use
our inductive assumption. Since B5# Ba5"b" , we have shown Ba5"b" B5" B. Now assume
Ba5"b; B5" B; for some ; 5. Then B5" B;" B5" B; B Ba5"b; B [inductive
a5"b;" a5"ba;"b
assumption] B B B .
Similarly, we can show aB+ b, B+, . First, notice aB" b B"" . Now assume aB8 b7 B87 for all 7 8 and
"

8 5 for some 5 . Then inductively we show aB8" b Ba8"b7 for all 7 8 ". First, aB8" b
7 "

Ba8"b" . Now assume aB8" b Ba8"b5 for some 5 8. Then aB8" b aB8" b aB8" b [part (a)]
5 5" 5

Ba8"b5 Ba8"b Ba8"b5a8"b [part (a)] Ba8"ba5"b . 


(b) As in part (a), we can show this inductively. First, aB" b B" . Assume B5 B5 . Then
" "

Ba5"b Ba5"b" B" B5 B" B5 B" B B5 1.1.1(4)] B5" .


" " "

Hence, aB+ b" B+ in general. 


[Proposition

(c) Let + be any integer. Then B+! B!+ B+ B! B+ B+ B! , and B!+ B+! "+ aB! b "!
+

aB+ b! . Hence, part (a) is valid when + or , is zero. Otherwise, consider when + and , are negative. Then we
know Ba+,b B+ B, B, B+ by part (a). Then Ba+,b B, B+ and using part (b) and
" "

Proposition 1.1.1(4), this yields Baa+,bb aB+ b" B, Ba+b Ba,b so that B+, B+ B, . Now
"

without loss of generality assume + is positive and , is negative. Consider k+k k,k. Then + a+ ,b ,
with both parts positive. Hence, B+ B, Ba+,b, B, B+, B , B, B+, . Now assume k+k k,k. Then
+ a+ ,b , with ,, + , negative, and k,k k+ ,k, so by what we have just proved,
B+ B+, B, . Therefore, B+ B, B+, B, B, B+, . 

Problem 1.1.20 For B an element in K show that B and B" have the same order.

Assume kBk 8 . By part (b) of the previous exercise, aB" b B8 aB8 b"
8
Proof.
"" ". All that remains to be shown is that this is the least 8. Assume there is a 7 such that 7 8
and aB" b ". Then aB7 b" " so that aB7 b" B7 "" ". However, this would contradict the
7 "

assumption kBk 7. Hence, kB" k 8. Now assume kBk _. Suppose B" has finite order, 8. Then
aB" b aB8 b" " so again aB8 b" B8 a"" b ". However, this would mean B has finite order,
8 "

a contradiction. Therefore, kB" k _. 

Problem 1.1.21 Let K be a finite group and let B be an element of order 8. Prove that if 8 is odd, then
B aB# b for some 5 .
5

If 8 ", B is the identity so the problem is trivial. Otherwise, let 7 be such that
8 #7 ". Then by the previous exercise, aB" b " so that aB" b aB" b aB" b B" ".
Proof.
7 7 #7" #7

Multiplying by B on the right hand side, aB" b B B aB" b aB#7 b B. Then aB#7 b
#7 " #7 " " "

is justified because B+ # B # BB B+" B.


+ +"
This
3" 3"
Robert Krzyzanowski Solutions to Dummit and Foote Chapter 1

B#7 B" . But by the previous exercise, B#7 aB# b . Hence, B" aB# b so that aB" b B
7 7 "

aB# b7 " aB# b7 aB# b5 for 5 7. If we wish to have a positive 5 , let < be the least positive
integer such that <8 7. Then B aB# b "#< aB# b aB8 b#< aB# b aB# b aB# b
5 5 5 <8 5<8
.

Problem 1.1.22 If B and 1 are elements of the group K, prove that kBk k1" B1k. Deduce that k+,k k,+k
for all + , K.

Proof 1. First, we will show that for all 8 , a1" B1b 1" B8 1. Inductively, when 8 " we have
8

a1" B1b 1" B" 1. Otherwise, assume a1" B1b 1" B5 1. Then
" 5

a1" B1 b a1" B1b a1" B1b a1" B5 1ba1" B1b 1" B5 a11" bB1 1" B5 B1 1" B5" 1.
5" 5

Hence, a1" B1b 1" B8 1 for 8 . Then if kBk 8 we see a1" B1b 1" B8 1 1" 1 ". Now
8 8

assume there is a 7 such that 7 8 and a1" B1b ". Then 1" B7 1 " so left and right
7

multiplication by 1 and 1" , respectively, yields 11" B7 11" 11" so that B7 ". However, this
contradicts the assumption kBk 8. Now assume that kBk _. If k1" B1k 8 _ kBk, then

a1" B1b 1" B8 1 "


8

so again left and right multiplication by 1 and 1" , respectively, yields B8 ", contradicting kBk _.
Hence, k1" B1k kBk for all elements B 1 K. To see the last part, let B +, and 1 ," +. Then

k+,k ka,+" ba+,ba," +bk k,+k. 

Proof 2. As in Proof 1, we know a1" B1b 1" B8 1. First, we show k1" B1k kBk. This is obvious
8

when kBk _. If kBk 8, then


a1" B1b 1" B8 1 1" 1 ",
8

so that k1" B1k kBk for all B 1 K. To see the opposite direction, substitute 1" B1 for B and 1" for 1 in
the previous statement:

a1" b a1" B1b1" kBk k1" B1k.


"

Thus, kBk k1" B1k for all B 1 K. To see the last part of the problem, just notice ,+ +" a+,b+ (e.g., take
B +, and 1 +). 

Problem 1.1.23 Suppose B K and kBk 8 _. If 8 => for some positive integers = and >, prove that
kB= k >.

First, it is clear aB= b> B=> B8 ". Assume there is a 7 such that 7 > and
aB= b7 ". Then B=7 ", but =7 => 8, so this contradicts the fact kBk 8. 
Proof.

Problem 1.1.24 If + and , are commuting elements of K, prove that a+,b8 +8 ,8 for all 8 .

Inductively, a+,b" +" ," . Assume a+,b5 +5 ,5 . Then a+,b5" a+,b5 a+,b +5 ,5 +,
+5 +,5 , +5" ,5" . Notice the penultimate step is justified by commutativity of + and ,. Hence, a+,b8
Proof.

+8 ,8 for all 8 . When 8 !, a+,b! " +! ,! . Finally, when 8 , we know a+,b8 +8 ,8 by

This somewhat more elegant proof is due to Anssi Lahtinen from Stanford: math.stanford.edu/~lahtinen/math-120-f08/hw-
solutions/math-120-sol-01.pdf
Robert Krzyzanowski Solutions to Dummit and Foote Chapter 1

what we have just shown, but + and , commute so a+,b8 a,+b8 ,8 +8 . Then aa+,b8 b
"

a+,b8 a,8 +8 b" a+8 b" a,8 b" +8 ,8 . 


Problem 1.1.25 Prove that if B# " for all B K then K is abelian.

Proof 1. Let B C K. Then aBCb# " since BC K. Hence, BCBC " so BC C" B" . However,
notice that aCBb# " since CB K, so CBCB " and consequently CB B" C" . Then aBCbaCBb
C" B" B" C" ". This implies BC B" C" CB. Hence, BC CB for all B C K and by definition K
is abelian. 

Proof 2. Let B C K. Then aBCb# BCBC ". Multiplying by B on the left and C on the right and
noting that B# " and C# ", we obtain CB BC. 

Problem 1.1.26 Assume L is a nonempty subset of aK b which is closed under the binary operation on K
and is closed under inverses, i.e., for all 2 and 5 L , hk and 2" L . Prove that L is a group under the
operation restricted to L (such a subset is called a subgroup of K).

Proof. Closure is given. Associativity follows from the lemma at the beginning of this section's
solutions. For each 2 L , "K 2 2 "K 2. Hence, "L "K . Finally, for each 2 L , there is an
2" L (by our assumption of closure of inverses) such that 22" 2" 2 "K "L . Hence, L is a
group under the operation. 

Problem 1.1.27 Prove that if B is an element of the group K then eB8 l 8 f is a subgroup of K (called
the cyclic subgroup of K generated by B).

Proof. Call the set L eB8 l 8 f. Let 2 5 L . Then there are : ; such that 2 B; and
5 B: . Then 25 B; B: B;: (exercise 19). Since ; : , 25 L by definition. Therefore, L is
closed under the operation of K Finally, if 2" B; , then 22" B; B; " and 2" 2 B; B; ".
Since ; , 2" L by definition, so L is closed under inverses. By the previous exercise, L is a
subgroup of K. 

Problem 1.1.28 Let aE b and aF b be groups and let E F be their direct product. Verify all the
group axioms for E F : .
(a) prove that the associative law holds: for all a+3 ,3 b E F , 3 " # $,
a+" ," bca+# ,# ba+$ ,$ bd ca+" ," ba+# ,# bda+$ ,$ b,
(b) prove that a" "b is the identity of E F , and
(c) prove that the inverse of a+ ,b is a+" ," b.

Proof. (a) Let a+3 ,3 b E F with 3 " # $. Then

a+" ," bca+# ,# ba+$ ,$ bd a+" ," ba+# +$ ,# ,$ b a+" a+# +$ b ," a,# ,$ bb

aa+" +# b +$ a," ,# b ,$ b a+" +# ," ,# ba+$ ,$ b

ca+" ," ba+# ,# bda+$ ,$ b.

The intermediate step follows because +" a+# +$ b a+" +# b +$ and ," a,# ,$ b a," ,# b ,$ by the
fact associativity holds for these elements because E and F is a group. 
(b) Let a+ ,b E F . Then a" "ba+ ,b a" + " ,b a+ ,b a+ " , "b a+ ,ba" "b. 
Robert Krzyzanowski Solutions to Dummit and Foote Chapter 1

(c) Let a+ ,b E F . a+ ,ba+" ," b a+ +" , ," b a" "b a+" + ," ,b
a+" ," ba+ ,b. 
Then

Problem 1.1.29 Prove that E F is an abelian group if and only if both E and F are abelian.

Proof. Assume E F is abelian. Then for all +" +# E and ," ,# F , it is true a+" ," ba+# ,# b
a+# ,# ba+" ," b. However, a+" ," ba+# ,# b a+" +# ," ,# b and a+# ,# ba+" ," b a+# +" ,# ," b. But then
a+" +# ," ,# b a+# +" ,# ," b, and the components must be equal by definition, so that +" +# +# +" and
," ,# ,# ," . Hence, E and F are abelian. Now assume this. Let a+" ," b a+# ,# b E F with +" +# E
and ," ,# F . Then a+" ," ba+# ,# b a+" +# ," ,# b a+# +" ,# ," b a+# ,# ba+" ," b. The intermediate step
is justified since we assumed E and F are abelian. By definition, we now know E F is abelian. 

Problem 1.1.30 Prove that the elements a+ "b and a" ,b of E F commute and deduce the order of a+ ,b
is the least common multiple of k+k and k,k.

Proof. We see a+ "ba" ,b a+ " " ,b a" + , "b a" ,ba+ "b. The intermediate step is

Let k+k 8 and k,k 7. Then a+ "b8 a+8 "8 b (by a trivial inductive argument) and so a+ "b8 a" "b. If
justified because + E, , F , and E and F are groups so multiplying + or , by the identity is commutative.

there were a 5 with 5 8 such that a+ "b5 a" "b. then +5 " 5 +5 " a" "b so that +5 ",
contradicting the fact k+k 8. Hence, ka+ "bk 8 k+k. Similarly, ka" ,bk 7 k,k. Now let
# lcma7 8b with # !7 and # " 8 for ! " . Then a+ ,b# a+# ,# b +!7 ," 8
a+7 b! a,8 b" "! "" a" "b. Now assume there is a $ such that $ # and a+ ,b$ ". Then
+$ ,$ " so that +$ ,$ ". Assume 8 l $ so that $ :8 < for some : < e!f with
! < 8. Then +$ +:8< +:8 + < a+8 b: +< ": +< +< ". However, this again contradicts the fact
k+k 8 since < 8, so that 8 l $ . Similarly, 7 l $ . But then by definition $ lcma7 8b # , a
contradiction. Hence, k+ ,k lcma7 8b. 

Problem 1.1.31 Prove that any finite group K of even order contains an element of order #.

Proof. Let kKk #8. If there was an element of order #, say B, we would have B# " so that B B" .
Let L e1 K l 1 1" f. Clearly, " L . Furthermore, consider the following procedure. Let L! L
and define L3" by removing some pair e13 1"
3 f from L3 with 13 L3 ; that is L3" L3 e1 1 f. In each
"

L5 g for some 5 . But then kL5" k kL5 k # #, kL5# k kL5" k # %, etc., so that
iteration, we know 1 1" so that two elements are removed. Since L is finite (as K is finite), eventually

kL! k kL k #7 for some 7 . Therefore, L has an even number of elements. Since " L ,
kL k kKk. It is impossible that kL k kKk " since kKk 1 is odd. Hence, kL k kKk ". In other words,
KaL e"fb g. But this means there is some B K such that B " with B B" (B L by definition).
That is, kBk #. 

Problem 1.1.32 If B is an element of finite order 8 in K, prove that the elements " B B# B8" are all
distinct. Deduce that kBk kKk.

Proof. Assume B3 B4 for some 3 4 (! 3 4 8). Without loss of generality, let 3 4. Then
B !, but 3 4 8, so this contradicts the fact kBk 8. Hence, " B B# B8" are all distinct. There
34

are 8 of these elements, so kKk 8 kBk. 

Problem 1.1.33 Let B be an element of finite order 8 in K.


(a) Prove that if 8 is odd then B3 B3 for all 3 " # 8 "
Robert Krzyzanowski Solutions to Dummit and Foote Chapter 1

(b) Prove that if 8 #5 and " 3 8 then B3 B3 if and only if 3 5 .

Proof. (a) If 8 " there is nothing to prove so assume 8 ". Assume B3 B3 for some
" 3 8 ". Then B3 B3 " so that B#3 ". Clearly, #3 8 since #3 is even and 8 is odd. If #3 8 there
would thus be a contradiction (since kBk 8 #3). Since 3 8, we then know 8 #3 #8 so that
! #3 8 8. However, B#3 Ba#38b8 B#38 B8 B#38 ". In other words, we found a positive
integer less than 8 such that B to the power of that integer is ". But this contradicts the fact kBk 8. Hence,
B3 B3 for all " 3 8 ". 
(b) Let B3 B3 for " 3 8 and assume 3 5 . Then B3 B3 " so that B#3 ". By assumption, #3 8. If
#3 8 there would thus be a contradiction (since kBk 8 #3). Since 3 8, we then know 8 #3 #8 so
that ! #3 8 8. However, B#3 Ba#38b8 B#38 B8 B#38 ". In other words, we found a positive
integer less than 8 such that B to the power of that integer is ". But this contradicts the fact kBk 8. Hence,
3 5 . Now assume 3 5 . Since " B8 B#5 B5 B5 , we have B5 B5 , or B3 B3 . 

Problem 1.1.34 If B is an element of infinite order in K, prove that the elements B8 , 8 are all distinct.

Proof. Assume B3 B4 for some 3 4 with 3 4. Then B34 ". If 3 4 !, this contradicts the
assumption B has infinite order. If 3 4 !, aB34 b "" " so that Ba34b B43 ". Then 4 3 !,
"

but again this contradicts the assumption B has infinite order. Hence, B3 B4 for all 3 4 with 3 4; that
is, the elements B8 , 8 are all distinct..

integral power of B equals one of the elements in the set e" B B# B8" f (so these are all the distinct
Problem 1.1.35 If B is an element of finite order 8 in K, use the Division Algorithm to show that any

elements of the cyclic subgroup of K generated by B).

Proof. Let 5 . Then 5 ;8 < for some ; , < with ! < 8 by the Division Algorithm.
Hence,
B5 B;8< B;8 B< aB8 b; B< "; B< B<
with ! < 8 as required. 

Problem 1.1.36 Assume K e" + , - f is a group of order % with identity ". Assume also that K has no
elements of order %. Use the cancellation laws to show that there is a unique group table for K. Deduce that
K is abelian.

Proof. Assume there are two group tables Q" and Q# with the same rows and colums both

such that Q" aB34 b Q# aB34 b where Q5 aB34 b is the 34th entry of group table (matrix) Q5 .
"representing" " + , and - , in that order (so that 1# +, 1$ ,, and 1% - ). Now assume there is some 3 4
Robert Krzyzanowski Solutions to Dummit and Foote Chapter 1

Chapter 1.2
Problem 1.2.1 Compute the order of each of the elements in the following groups:
(a) H' (b) D) (c) H"! .

Solution. This problem is trivial. The solution for each group is

(a) The orders are l"l ", l<l $ l<# l $, k=k k=<k k=<# k #.

(b) The orders are k"k ", k<k k<$ k %, k<# k k=k k=<k k=<# k k=<$ k #.

(c) The orders are k"k ", k<k k<# k k<$ k k<% k &, k=k k=<k k=<# k k=<$ k k=<% k #.

Problem 1.2.2 Use the generators and relations < = l <8 =# ", <= =<" to show that if B is any
element of H#8 , which is not a power of <, then <B B<" .

If B is not a power of <, B =<3 for some ! 3 8. Then <= =<" so that a<=b<3 =<" <3 ,
or in other words, <a=<3 b a=<3 b<" . 
Proof.

Problem 1.2.3 Use the generators and relations above to show that every element of H#8 which is not a
power of < has order #. Deduce that H#8 is generated by the two elements = and =<, both of which have order
#.

If B is not a power of <, B =<3 for some ! 3 8. But then <3 = <3" a<=b <3" a=<" b by
one of the relations of H#8 , and applying this 3 " times, <3 = =a<" b =<3 , so that <3 = a<3 =b and
Proof.
3 "

hence k<3 =k #. Since =a=<b <, = =< = =< < < = H#8 , H#8 is generated by = and =<. 

Problem 1.2.5 If 8 is odd and 8 $, show that the identity is the only element of H#8 which commutes with
all elements of H#8 .

Proof. Let 8 be odd and greater than ". No <3 commutes with all elements for some " 3 8. If it
did, it would particularly for = giving <3 = =<3 , but our knowledge that <3 = =<3 would then mean
=<3 =<3 , or <3 <3 or <#3 ". Since " 3 8, we know # #3 #8 so that the former statement can
only be true when #3 8. The fact 8 is odd gives a contradiction. Finally, assume =<3 commutes with all
elements for some " 3 8. But then

a=<3 b< <a=<3 b =<3" <=<3 =a=<3" b =a<=<3 b <3" a=<ba=<3 b a<" =ba=<3 b <3"

which implies <a3"ba3"b <# " again giving a contradiction. 

Problem 1.2.6 Let B and C be elements of order # is any group K. Prove that if > BC then >B B>" (so
that if 8 kBCk _ then B > satisfy the same relations in K as = < do in H#8 ).

The elements B and C are of order # so that B" B and C" C. Then >" aBCb"
C" B" CB so that >B aBCbB BaCBb B>" . 
Proof.

Problem 1.2.7 Show that + , l +# ,# a+,b8 " gives a presentation for H#8 in terms of the two
generators + = and , =< of order # computed in Exercise $ above.

Proof. Notice in the usual presentation of H#8 , +, =a=<b <. Hence, we need to show that
Robert Krzyzanowski Solutions to Dummit and Foote Chapter 1

+ , l +# ,# a+,b8 " +, + l a+,b8 +# ", a+,b+ +a+,b"

meaning of +, as "+ times ," can be dropped). Notice , +# , +a+,b. Hence, any element generated by +
because the latter is the presentation of H#8 with +, < and + = (considered as literal strings, where the

and , is also generated by + and +,, and conversely any element generated by +, and + is also generated by +
and ,.
To show the relations of the latter are implied by the former, notice we already have a+,b8 +# ". To
show the final relation, we need to prove a+,b+ +a+,b" but by the cancellation law this amounts to
,+ a+,b" . This is trivial since a+,ba,+b +,# + +# " so indeed ,+ a+,b" . 

Problem 1.2.8 Find the order of the cyclic subgroup of H#8 generated by <.

Solution. We know <! a "b, <" <# <8" are all distinct elements of H#8 with <8 ". Furthermore,

<5 <;8: a<8 b; <: "; <: <: . Hence, we know any <5 is equivalent to one of the aforementioned
we know if 5 then by the division algorithm 5 ;8 : for some ; : with ! : 8. Hence,

elements. Thus, <5 l 5 (the cyclic subgroup of H#8 generated by <) has 8 distinct elements given by
<! <" <8" so that by definition its order is 8. 

In each of Exercises 9 to 13 you can find the order of the group of rigid motions in $ of the given Platonic
solid by following the proof for order of H#8 : find the number of positions to which an adjacent pair of
vertices can be sent. Alternatively, you can find the number of places to which a given face may be sent and,
once a face is fixed, the number of positions to which a vertex on that face may be sent.

Problem 1.2.9 Let K be the group of rigid motions in $ of a tetrahedron. Show that kKk "#.

Proof. A rigid motion of the tetrahedron is taking a copy of the tetrahedron, moving this copy in any
fashion in 3-space, and then placing the copy back on the original 8-gon so it exactly covers it. Notice this
rigid motion is a symmetry (and notice we could get more symmetries by moving through %-space, which can
also be made into a group). We can describe these symmetries by choosing some labelling of the % vertices

5 e" # 8f e" # 8f where if the symmetry = places vertex 3 in the position where vertex 4 was
numerically from " to %. Then each symmetry = can be described uniquely by the corresponding permutation

originally, then 5 is the corresponding permutation sending 3 to 4. These symmetries can be made into a
group K as follows. Define => for = > K to be the symmetry obtained by applying > and then = to the
tetrahedron. If = > effect the permutations 5 7 respectively on the vertices, then => will effect 5 7 . The
binary operation of K is associative since composition of functions is associative. The identity of K is the
identity symmetry (effecting the identity permutation), and the inverse of = K is the symmetry that reverses
all rigid motions of = (so that if = effects 5, =" effects 5" ). This shows we can indeed make a group out of
the rigid motions of a tetrahedron, just like with H#8 .
To show kKk "#, notice for any vertex 3 and some adjacent vertex 4, there is are $ symmetries which
send vertex " into position 3. A particular symmetry is obtained by sending the vertex 4 to any of the $
adjacent vertices to 3. Hence, there are % $ "# possible such symmetries (since a tetrahedron has four
vertices). These symmetries are the only symmetries of the tetrahedron, since a rigid motion in $ is
completely determined by where it positions three given points (apply this to the vertices 3, 4, and the unique
vertex adjacent to these two). 

Problem 1.2.10 Let K be the group of rigid motions in $ of a cube. Show that kKk #%.

Proof. As in Problem 1.2.9, it can be verified that K is indeed a group. To show kKk #%, notice for
any vertex 3 and some adjacent vertex 4, there is are $ symmetries which send vertex " into position 3. A
Robert Krzyzanowski Solutions to Dummit and Foote Chapter 1

particular symmetry is obtained by sending the vertex 4 to any of the $ adjacent vertices to 3. Hence, there are
) $ #% possible such symmetries (since a cube has eight vertices). These symmetries are the only
symmetries of the cube, since a rigid motion in $ is completely determined by where it positions three given
points (apply this to the vertices 3, 4, and the unique vertex adjacent to these two). 

Problem 1.2.11 Let K be the group of rigid motions in $ of a octahedron. Show that kKk #%.

Proof. As in Problem 1.2.9, it can be verified that K is indeed a group. To show kKk #%, notice for
any vertex 3 and some adjacent vertex 4, there is are % symmetries which send vertex " into position 3. A
particular symmetry is obtained by sending the vertex 4 to any of the % adjacent vertices to 3. Hence, there are
' % #% possible such symmetries (since an octahedron has six vertices). These symmetries are the only
symmetries of the octahedron, since a rigid motion in $ is completely determined by where it positions three
given points (apply this to the vertices 3, 4, and the unique vertex adjacent to these two). 

Problem 1.2.12 Let K be the group of rigid motions in $ of a dodecahedron. Show that kKk '!.

Proof. As in Problem 1.2.9, it can be verified that K is indeed a group. To show kKk '!, notice for
any vertex 3 and some adjacent vertex 4, there is are $ symmetries which send vertex " into position 3. A
particular symmetry is obtained by sending the vertex 4 to any of the $ adjacent vertices to 3. Hence, there are
#! $ '! possible such symmetries (since an octahedron has twenty vertices). These symmetries are the
only symmetries of the dodecahedron, since a rigid motion in $ is completely determined by where it
positions three given points (apply this to the vertices 3, 4, and the unique vertex adjacent to these two). 

Problem 1.2.13 Let K be the group of rigid motions in $ of an icosahedron. Show that kKk '!.

Proof. As in Problem 1.2.9, it can be verified that K is indeed a group. To show kKk #%, notice for
any vertex 3 and some adjacent vertex 4, there is are & symmetries which send vertex " into position 3. A
particular symmetry is obtained by sending the vertex 4 to any of the & adjacent vertices to 3. Hence, there are
"# & '! possible such symmetries (since an icosahedronhas twelve vertices). These symmetries are the
only symmetries of the icosahedron, since a rigid motion in $ is completely determined by where it
positions three given points (apply this to the vertices 3, 4, and the unique vertex adjacent to these two). 

Problem 1.2.14 Find a set of generators for .

consider e"f. First, notice ! " a"b. Furthermore, for 5 ,


Solution. Let the group operation be and an inverse of an element 5 be written 5. It suffices to

5 a
"b a"b a"b,
5 times
and for 5 ,
5
" " " .
5 times

its inverse). By definition, e"f is a set of generators for , that is ". 


Hence, each element in can be represented as a sum of "'s and "'s (the group operation applied to "'s and

Problem 1.2.15 Find a set of generators and relations for 8.


Robert Krzyzanowski Solutions to Dummit and Foote Chapter 1

Solution. Let + 8. Then ! " " and we for any other element
+ "
"
+w times
where +w with ! + 8 so that +w +. Hence, " generates 8. If for 8 we define 8+ to be !
if 8 !, the sum + + + taken 8 times if 8 is positive, and the sum +" +" taken 8 times
if 8 is negative, then the only relation we need for 8 is 8" !. Notice ! " " " a8 "b " must all
be distinct elements so we know the order of a group with this generator and relation must be at least 8.
Furthermore, these elements must be the only elements of such a group. For any element +, the group being

! ; 8 such that 5 :8 ; , so that + a:8 ; b" a:8b" ;" :8" ;" : ! ;" ;"
generated by " means there is a 5 such that + 5". By the division algorithm, there are : ; with

a8 "b ". Hence, there are exactly 8 elements in such a group. In conclusion, the only such group is 8
where 8" ! was given by our sole relation. In other words, any element + must be one of ! " " "

so that
8 " l 8" !. 

Problem 1.2.16 Show that the group B" C" l B#" C"# aB" C" b# " is the dihedral group H% (where B"
may be replaced by the letter < and C" by =).

Proof. Recall the usual presentation of H#8 ,


< = l <8 =# ", <= =<" .
Then a presentation of H% H## is < = l <# =# ", <= =<" . If we replace B" by < and C" by =, we
want to show this presentation is equivalent to < = l <# =# ", a<=b# ". The first relation in this
presentation is the same as the first in the usual presentation of H% , so we merely need to show a<=b# "
implies the relation <= =<" . This is clear, since a<=b# " means <=<= " so that <=<==" <" =" <"
and on the left-hand side ==" cancel and then <<" cancel so we obtain <= =" <" =<" (since =# "
gives = =" ). 

Problem 1.2.17 Let \#8 be the group whose presentation is displayed in (1.2).
(a) Show that if 8 $5 , then \#8 has order ', and it has the same generators and relations as H' when
B is replaced by < and C by =.
(b) Show that if a$ 8b ", then B satisfies the additional relation: B ". In this case deduce that \#8
has order #.

Proof. The presentation in question is given by


\#8 B C l B8 C# " BC CB# .
(a) Assume 8 $5 . As has been shown in the discussion in the book, \#8 is of order at most ' and a
"hidden" relation is B$ ". Notice because 8 $5 the first relation B8 " can be replaced by B$ " since
the former gives us no new information. Hence, B# B" so that BC CB" . In other words, given this
knowledge about 8 we can rewrite the presentation,
\#8 B C l B$ C# " BC CB" .
However, notice if B and C are replaced by < and = respectively, this is exactly the canonical presentation of
H' . Therefore, a group satisfying the presentation for \#8 with at least ' elements exists, and so \#8 must be
of order exactly '. 
Robert Krzyzanowski Solutions to Dummit and Foote Chapter 1

(b) Assume a$ 8b " so that 8 $5 " for some 5 !. Then as in part (a) we know B$ ", so that
B8 " gives " B8 B$5" aB$ b B" "5 B B. Hence, B ", and since any element must be of the
5

form C5 B3 with ! 3 8 " and 5 ! or ". This reduces to just C5 , so \#8 has at most two possible
elements, C and the identity. Additionally, the representation reduces to C l C# ". Notice from Problem
1.2.15, this is exactly the representation for # (with C# " written # C !), so there is such a group
\#8 with exactly # elements. Hence, this presentation must give a group with order #. 

Problem 1.2.18 Let ] be the group whose presentation is displayed in (1.3).


(a) Show that @# @" .
(b) Show that @ commutes with ?$ .
(c) Show that @ commutes with ?.
(d) Show that ?@ ".
(e) Show that ? ", deduce that @ ", and conclude that ] ".

Proof. Let ] be the group whose presentation is given by


] ? @ l ?% @$ " ?@ @# ?# .
Then since @$ ", we know @# @ " so that @# @" . We now show @?$ ?$ @. This is equivalent to
showing @" ?$ @ ?$ , or ?$ @# ?$ @. This we can show because
@# ?$ @ a@# ?# ba?@b a?@ba@# ?# b ?@$ ?# ?$ .
Hence @ commutes with ?$ . Next, notice ?* ?) ? a?% b ? "# ? ?. Then
#

@? @?* a@?$ b?' a?$ @b?' ?$ a@?$ b?$ ?$ a?$ @b?$ ?' a@?$ b ?' a?$ @b ?* @ ?@.
Hence, @ also commutes with ?. Then the relation ?@ @# ?# implies @?@ ?# so that ?@# ?# and so
?$ @# ", or ?" @" " (since ?% " implues ?$ ?" ). Hence, @? ?@ ". Then " a?@b$
?$ @$ ?" so that ? ". Finally, the last relation is reduced to @ @# so that the cancellation law gives
@ ". Hence, the only element in this group is the identity, so it must be the trivial group; that is, ] ". 
Robert Krzyzanowski Solutions to Dummit and Foote Chapter 1

Chapter 1.3
Problem 1.3.1 Let 5 be the permutation

"$ #% $& %# &"

and let 7 be the permutation

"& #$ $# %% & ".

Find the cycle decompositions of each of the following permutations: 5 7 5# 57 75 and 7 # 5 .

Solution. First, notice 5 a" $ &ba# %b and 7 a" &ba# $b. Hence,
5# 5 5 a" $ &ba# %b a" $ &ba# %b a" & $b
57 5 7 a" $ &ba# %b a" &ba# $b a& $ % #b.
75 7 5 a" &ba# $b a" $ &ba# %b a" # % $b.
7 # 5 7 7 5 a" &ba# $b a" &ba# $b a" $ &ba# %b a" $ &ba# %b.

Problem 1.3.2 Let 5 be the permutation


" "$ # # $ "& % "% & "! ' ' ( "# )$ *% "! "
"" ( "# * "$ & "% "" "& )

and let 7 be the permutation


" "% # * $ "! % # & "# ' ' (& ) "" * "& "! $
"" ) "# ( "$ % "% " "& "$.
Find the cycle decomposition of the following permutations: 5 7 5# 57 75 and 7 # 5 .

Solution. First, notice


5 a" "$ & "!ba$ "& )ba% "% "" ( "# *b and 7 a" "%ba# * "& "$ %ba$ "!ba& "# (ba) ""b.
Hence,
5# a" &ba$ ) "&ba% "" "#ba( * "%ba"! "$b 57 a" "" $ba# %ba& * ) ( "! "&ba"$ "%b
75 a" %ba# *ba$ "$ "# "& "" &ba) "! "%b 7 # 5 a" # "& ) $ % "% "" "# "$ ( & "!b
Notice 7 # 5 is one big cycle that fixes ' and *. 

Problem 1.3.3 For each of the permutations whose cycle decompositions were computed in the precesing
two exercises, compute its order.

Solution. The orders can either be computer directly (a very laborious process), or a quick mental
inspection which will later be proved in Problem 1.3.15 yields

k5k k"#k, k7 k k$!k k5# k k57 k k57 k k'k, and k7 # 5 k "$


for the previous exercise and
k5k k7 # 5k ', k7 k #, k5# k k57 k k75 k %.
Robert Krzyzanowski Solutions to Dummit and Foote Chapter 1

for the first exercise. 

Problem 1.3.4 Compute the order of each of the elements in the following groups (a) W$ (b) W% .

Solution. As in the previous problem, in W$


k"k ", ka# $bk ka" $bk ka" $bk #, ka" # $bk ka" $ #bk $
and in W% the above permutations (the ones that fix %) all have the same orders, and
k a " % b k k a # % b k k a $ % b k #,
ka" # %bk ka# " %bk ka" $ %bk ka$ " %bk ka# $ %bk ka$ # %bk $,
ka" # $ %bk ka# " $ %bk ka" $ # %bk ka# $ " %bk ka$ " # %bk ka$ # " %bk %, and
ka" #ba$ %bk ka" $ba# %bk ka" %ba# $bk #.
The author of the book probably intended these elements to be written the permutations to be written
explicitly (e.g., define 53 for each of the 8x permutations in W8 ), but finding the cycle decomposition is a
fairly trivial process for these elements, so they were written as such, and hence they are indeed all unique
elements. Note doing this makes Problems 1.3.6 and 1.3.7 trivial.

Problem 1.3.5 Find the order of a" "# ) "! %ba# "$ba& "" (ba' *b.

Proof. As in Problem 1.3.3, ka" "# ) "! %ba# "$ba& "" (ba' *bk $!. 

Problem 1.3.6 Write out the cycle decomposition of each element of order % in W% .

Proof. See Problem 1.3.4. 

Problem 1.3.7 Write out the cycle decomposition of each element of order # in W% .

Proof. See Problem 1.3.4. 

Problem 1.3.8 Prove that if H e" # $ f then WH is an infinite group (do not say _x _).

Consider permutations of the form 53 a" 3b, that is, those for which " and 3 are exchanged
and all other elements are fixed. There are an (countably) infinite such 53 and since e53 f WH , ke53 fk kWH k
Proof.

so that WH is an infinite group. 

Problem 1.3.9
(a) Let 5 be the "#-cycle a" # $ % & ' ( ) * "! "" "#b. For which positive integers 3 is 53 also a "#-cycle?
(b) Let 7 be the )-cycle a" # $ % & ' ( )b For which positive integers 3 is 7 3 also an )-cycle?
(c) Let = be the "%-cycle a" # $ % & ' ( ) * "! "" "# "$ "%b For which positive integers 3 is =3 also a "%-
cycle?

only if a8 3b ". Hence, (a) for a "#-cycle, 53 is also a "#-cycle if and only if 3 e" & ( ""f, (b) for an )-
Proof. It is an obvious result (proved in Problem 1.3.11) that for an 8-cycle 5, 53 is an 8-cycle if and

cycle, 53 is also an )-cycle if and only if 3 e" $ & (f, and (c) for a "%-cycle, 53 is also a "%-cycle if and
only if 3 e" $ & * "" "$f 

Problem 1.3.10 Prove that if 5 is the 7-cycle a+" +# +7 b, then for all 3 e" # 7f, 53 a+5 b +53 ,
where 5 3 is replaced by its least residue mod 7 when 5 3 7. Deduce that k5k 7.
Robert Krzyzanowski Solutions to Dummit and Foote Chapter 1

Proof. We proceed inductively. First, notice 5a+5 b +5" where 5 " is considered mod 7. This is
because if 5 7, then 5 assigns +5 to the element in the cycle adjacent on the right to +5 , i.e., +5" . If

53 a+5 b +53 for some 3 7. Then 53" a+5 b 5 53 a+5 b 5 a5 3 a+5 bb 5 a+53 b. Again, if 5 3 7,
5 7, then 5 assigns +5 to the first element in the cycle, i.e., +" +7" +5" . Now assume that

assigns +53 to the first element in the cycle, i.e., +" +7" +53" . Hence, 53" a+5 b 5 a+53 b
then 5 assigns +53 to the element in the cycle adjacent on the right to +53 , i.e., +53" . If 5 3 7, then 5

+5a3"b . Therefore, for any 3 e" # 7f, 53 a+5 b +53 . Since


5! a+5 b +5! 5" a+5 b +5" 57" a+5 b +5a7"b
are all distinct elements, k5k 7. However, 57 a+5 b +57 +5 then implies k5 k 7. 

Problem 1.3.11 Let 5 be the 7-cycle a" # 7b. Show that 53 is also an 7-cycle if and only if 3 is
relatively prime to 7.

Assume 3 7. By the previous exercise, 53 a+5 b +53 and the inductive argument is easily
extended to 3 7 by recognizing +5 for 5 define equivalence classes on e+" +7 f with +5 +3 for
Proof.

" 3 7 if and only if 5 3 (mod 7). Hence, 583 a+5 b +583 . Notice 583 a+5 b +5 for all +5 (i.e., 53 is
by definition an 8-cycle) if and only if 5 5 83 amod 7b for all 5 e" # 7f which holds if and only
if 83 ! amod 7b. This is obvious for 3 7 so assume 3 7. If a3 7b ", the lowest such integer 8 is 7
since any other integer 8 7 must satisfy 7 l 83 and hence also 7 l 3 (by the fact 8 is relatively prime to 7

other direction follows when we show that if the lowest integer 8 such that 83 ! amod 7b is 8 7, then
and so they can have no common factors), which is impossible since 3 7. Hence 53 is an 7-cycle. The

a3 7b ". Assume this hypothesis. Then if a3 7b ", i.e., if b. " such that . l 3 and . l 7, this would
mean that 7. 3. so that a7. b3 7a3. b ! amod 7b would contradict that 7 is the lowest such
integer for 8 (since 7. 7). Hence a3 7b ".

Since 57 a+5 b +57 +5 , we have 57 ". Then


To finish the argument for 3 7, notice by the division algorithm 3 ;7 : for ; : , ! : 7.

53 a+5 b 5;7: a+5 b 5;7 5: a+5 b a57 b; 5: a+5 b a"b; 5: a+5 b 5: a+5 b
for which we have already proven the statement. 

Problem 1.3.12
(a) If 7 a" #ba$ %ba& 'ba( )ba* "!b determine whether there is an 8-cycle 5 a8 "!b with 7 55 for
some integer 5 .
(a) If 7 a" #ba$ % &b determine whether there is an 8-cycle 5 a8 &b with 7 55 for some integer 5 .

Proof. (a) Yes, namely 5 a" $ & ( * # % ' ) "!b for 5 &. This can be deduced by realizing each of
the elements in each #-cycle must be "!# & elements apart in a "!-cycle so that they can all "reach each
other" while passing over the elements in the other #-cycles.
(b) No. We can attempt to construct such a 5. If there was such a 5 a+" +# +8 b, then assume without loss

to fix e+3 l +3 &f through 55 (as 7 does), we would need 8 l 5 . If this were false, take +3 to be such that
of generality (we can cyclically permute the elements in the cycle until we get) +" ". If 8 &, then in order

+3 ', and then (decomposing 5 as usual with the division algorithm for ! < 8)
55 a+3 b 5;8< a+3 b a58 b; 5< a+3 b a"b; 5 < a+3 b 5 < a+3 b +3< +3
so that ' would not be fixed. However, then 8 l 5 would imply for any 3 e" 8f,
Robert Krzyzanowski Solutions to Dummit and Foote Chapter 1

55 a+3 b 5;8 a+3 b a58 b; a+3 b "; a+3 b +3 ,


that is, any element in the 8-cycle would be fixed. Hence, 8 &, meaning 5 a" +# +$ +% +& b. However, this
does not work either, since we need 55 a+" b 55 a"b # +3 (for some # 3 &) and 55 a+3 b +5 a#b
" +" , so that 55 55 a+" b +" , and hence 5 #5 a+" b +"#5 +" meaning " #5 " amod &b, or
#5 ! amod &b. However, a# &b " so we would need & l 5. However, then 55 a+" b +"5 +" ,
contradicting our assumption 55 a+" b +3 . Hence, there is no such 8-cycle 5 for 8 & (and indeed, none for
any 8). 

Problem 1.3.13 Show that an element has order # in W8 if and only if its cycle decomposition is a product
of commuting #-cycles.

Assume an element 5 has order # If its cycle decomposition had an <-cycle a+" +< b for
< # we would attain a contradiction as follows. Since 5# a+" b +"# +$ +" (because this <-cycle has
Proof.

at least $ elements), 5# would not fix +" so that by definition the order of 5 #. Hence its cycle
decomposition is a product of commuting #-cycles.

Conversely, assume 5 a+" ," ba+< ,< b. Then for " 3 <, 5# a+3 b 5 a5a+3 bb 5 a,3 b 5 a+3 b and
similarly 5# a,3 b 5 a5a,3 bb 5 a+3 b ,3 and hence 5 # " so that k5 k #. 

Problem 1.3.14 Let : be a prime. Show that an element has order : in W8 if and only if its cycle
decomposition is a product of commuting :-cycles. Show by an explicit example that this need not be the case
if : is not prime.

Assume 5 W8 is a product of commuting :-cycles 53 , 5 # 53 . By Problem 1.3.10, notice


k53 k : for each 3 so that 53: ". By commutativity of the cycles,
Proof.

5: a# 53 b: # 53: # " ".


Notice the order of 5ak5k =b could not be less than :, because by the disjointness of the :-cycles this would
imply that 53= " for some 3 with = : (in fact, for all 3), which is a contradiction. Hence k5k :.
Conversely, let k5k :. Assume 5 has a cycle decomposition with an <-cycle 7 a+" +< b for < :
(with < "). Then we need 5: a+" b 7 : a+" b +": +" . This would mean " : " amod <b, so that
: ! amod <b. This is clearly false when < :, so consider < :. The condition holds when < l :, but this
would imply either < : or < ", a contradiction since " < :. Hence 5 has a cycle decomposition with
only :-cycles. Take the unique cycle decomposition and its :-cycles will be disjoint, so that they commute.
This need not be true when : is not prime. The element a" $ # %b W% has order # but is a product of
commuting %-cycles. 

Problem 1.3.15 Prove that the order of an element in W8 equals the least common multiple of the lengths of
the cycles in its cycle decomposition.

Assume the order of an element 5 W8 is smaller, with 5 #3 53 for 53 disjoint 83 -cycles.


Then there is such an 3 so that 83 l53 l l 8 k5k (the former equality is by Problem 1.3.10), meaning
Proof.

8 ;83 < for ; < with ! < 83 . First, notice it is necessary 538 58 5383 " (since all elements
fixed by 58 must be fixed by 538 , and also the order of 53 is 83 ). If 53 a+" +83 b, then
538 a+" b 53;83 < a+" b a5383 b; 5< a+" b 5 < a+" b +"< +"
since ! < =3 . This contradicts that k5k = (since one of its cycles does not fix +" when applied to it =
times). If 8 lcme83 k53 k 53 a 83 -cycle in the cycle decomposition of 5f, then 83 l 8 (say 8 ;3 83 ) so
Robert Krzyzanowski Solutions to Dummit and Foote Chapter 1

58 a#3 53 b8 #3 538 #3 53 ;3 83 #3 a5383 b 3 #3 a"b;3 ".


;

Hence k5k 8. 

Problem 1.3.16 Show that if 8 7 then the number of 7-cycles in W8 is given by


8a8"ba8#ba87"b
7 .

Let 5 a+" +7 b. There are 8 ways of choosing +" , 8 " ways of choosing +# (since +" can
not appear again), etc., so there are in total 8a8 "ba8 #ba8 7 "b possible 7-cycles. However, not
Proof.

all of these are unique, since cyclically permuting the elements of an 7-cycle is an equivalence relation. For
each 7-cycle, there are 7 such permutations, corresponding to +3 +3 , +3 +37 (that is, +3 +35
means the cycle's elements are shifted to the right by 5 spots, leaving the same cycle but a different
representation). Hence, there are in total
8a8"ba8#ba87"b
7

7-cycles in W8 . 

disjoint #-cycles is na8 "ba8 #ba8 $b).


Problem 1.3.17 Show that if 8 % then the number of permutations in W8 which are the product of two

The elements in question are of the form a+ ,ba- . b. By the previous problem, there are
8a8 "b# ways to form the first #-cycle. However, now we need - . e" 8f e+ ,f to satisfy that
Proof.

we have disjoint cycles. Since ke" 8f e+ ,fk ke" 8fk ke+ ,fk 8 #, there are 8 # ways of
picking - , and then 8 $ ways of picking . so that - . . Hence, there are a8 #ba8 $b ways in total.
However, if a- . b is a 2-cycle, a. - b is equivalent to it so we divide by #. In summary, there are:
8a8"b a8#ba8$b 8a8"ba8#ba8$b

different elements which are products of disjoint #-cycles. However, when a+ ,ba- . b is a #-cycle, by
# # %

commutativity of disjoint cycles a+ ,ba- . b a- . ba+ ,b. Hence, we have to divide by # again to obtain the
final number of unique products of two disjoint #-cycles:
8a8"ba8#ba8$b 8a8"ba8#ba8$b
% # ) . 

Problem 1.3.18 Find all numbers 8 such that W& contains an element of order 8.

Proof. The possible cycles in W& are # $ % and & cycles. Hence, unique permutations in W& can be
written as one of these, or a product of #- and #- or #- and $-cycles (no #- and %-, #- and &-, or %- and &-
cycles, since these would not be disjoint and hence not a valid cycle decomposition). Problem 1.3.15 gives
8 e" # $ % & lcma# #b lcma# $bf e" # $ % & 'f.

Problem 1.3.19 Find all numbers 8 such that W( contains an element of order 8.

Proof. The possible cycles in W( are # $ %, &, ', and ( cycles. Hence, unique permutations in W( can
be written as one of these, or a product of #- and #- #- and $-, 2- and 4-, 2- and 5-, $- and $-, $- and %-, cycles
(none of the other products of cycles, since these could not be disjoint and hence not a valid cycle
decomposition). Thus, by Problem 1.3.15,
8 e" # $ % & ' ( lcma# #b lcma# $b lcma# %b lcma# &b lcma$ $b lcma$ %bf e" # $ % & ' ( "! "#f.

Problem 1.3.20 Find a set of generators and relations for W$ .


Robert Krzyzanowski Solutions to Dummit and Foote Chapter 1

Proof. Notice a single generator would have to be of order '. However, by Problem 1.3.15 the only

this is impossible since that would require & distinct elements in e" # $f, and there only are $. Hence, W$ is
way to achieve this would be to have an element have a cycle decomposition of a 2- and a 3-cycle. However,

5 a" #b, 7 a" $b. Then 5# ", 75 a" # $b, 57 a" $ #b, and 575 a# $b, so indeed the elements
not generated by one element. Additionally, we cannot have commutative (disjoint) generators. Let

generate W$ . Then the relations 5# 7 # a57 b$ " suffice. It's easy to see the last relation implies
a57 b# 75, which means 57 75 and neither 57 nor 75 reduce to " 5 or 7 (by examining orders). This
gives us five elements, " 5 7 57 75. Furthermore, notice no element can have a 5# or 7 # since these would
just cancel out. Hence each element has to be a "chain" of single 5's and 7 's. Furthermore, a57 b# 75 gives
575 757 so that any chain of 5 and 7 's of length greater than $ can be reduced to a chain of length $ or

reduced. To verify this last one is unique, notice a575b# ", but it cannot be either of the other two elements
less. Hence the only chain of length $ is 575, and these are all possible elements since any others can be

of order # (5 or 7 ), because a575 b5 57 " and a575 b7 a757 b7 75 ". Hence, the elements
" 5 7 57 75 575 are all unique and are the only possible elements, so the group is indeed of order '. Of
course we constructed this presentation explicitly from W$ , W$ satisfies these generators and relations:
W$ + , l +# ,# a+,b$ ".
Notice omitting the last relation would give us the infinite dihedral group Dih_ (the group of symmetries
generated by a reflection and rotation, where the rotation is not a rational multiple of a full rotation; i.e., the
group of symmetries of a circle), where +, is a product of two elements of finite order (with torsion) but itself
is of infinite order (torsion-free). 
Robert Krzyzanowski Solutions to Dummit and Foote Chapter 1

Chapter 1.4
Problem 1.4.1 Prove that kKP# a# bk '.

Elements must be of the form E


- .
+ ,
Proof. with each + , - . equal to 0 or ". This matrix is
invertible (in KP# a# b) if and only if det E +. ,- !. That is, we need +. ,- . Notice if BC " if
and only if B C " for B C # Hence, if + " and . ", then either a- ,b a! !b, a! "b, or a" !b.
On the other hand, if a+ . b a! !b, a! "b, or a" !b we would need a- . b a" "b. Hence we have six such
possible a+ , - . b, so that kKP# a# bk '. 

Problem 1.4.2 Write out all the elements of KP# a# b and compute the order of each element.

Solution.

! ", #, and
"
" ! " " " ! ! " " " ! "
$. 
" ! " " " " " " ! "

Problem 1.4.3 Show that KP# a# b is non-abelian.

Proof.

! " " " " " " ! " " ! "


" " " ! ! " " " " ! " "
.

Problem 1.4.4 Show that if 8 is not prime then 8 is not a field.

Assume 8 +, for some + , e"f. Then aa8b b is not a group because the
operation is not well-defined: + , ! a8b . Hence it is not a field. 
Proof.

Problem 1.4.5 Show that KP8 aJ b is a finite group if and only if J has a finite number of elements.

Proof. Assume KP8 aJ b is finite and J is not. This is a contradiction because for each + J ,

! +"
+ !

has non-zero determinant ( "), and each of these elements is distinct, so that KP8 aJ b would have infinite
elements.
Take J to have finitely many elements, say <. Then E KP8 aJ b must be of the form

- .
+ ,
with + , - . J ,

and there are hence at most <% distinct elements in KP 8 aJ b (< choices for + < choices for , ...) . 

Problem 1.4.6 If kJ k ; is finite prove that kKP8 aJ bk ; 8 .


#

An element in KP8 aJ b is a matrix with 8 rows and 8 columns, with each spot in the matrix
taking one of ; possible values (since kJ k ; and the element must be in J ). Hence, an upper bound for the
Proof.

unique number of distinct elements in KP8 aJ b is (taking +34 J to be the 3th row and 4th column):
Robert Krzyzanowski Solutions to Dummit and Foote Chapter 1

# # a; choices for +34 b ; 8# .


8 8

3" 4"

To show kKP8 aJ bk ; 8#
it then suffices to find one element of this form not in KP8 aJ b, e.g.,

! !
! !
.

Problem 1.4.7 Let : be a prime. Prove that the order of KP# a: b is :% :$ :# :.

Proof. As in Problem 1.4.5, KP# a: b has at most :% elements (all possible matrices, invertible and
non-invertible). From linear algebra, we know a matrix is invertible if and only if its rows are linearly
independent. Hence, matrices of the following form are NOT invertible:

-+ and
,
+ , -+ -,
(1)
-, +

Notice the second form of matrix can almost always be constructed by the first. For example, if we want to
start with . / in the bottom row and look at multiples of it for the first row, we could let + -. and , -/
so that

. "
- /
-. -/ + ,
" .
/ - +
However, notice - has a multiplicative inverse if and only if - : if and only if - !. Hence, the second

non-invertible matrices in KP# a: b: elements are of the form


form of matrix in (1) can be expressible in the first if and only if - !. This gives us a classification of all

-+ or
/
+ , ! !
with + , - . / J and + and , not both !.
-, .
Now it remains to count how many possible options there are. There are :# " choices for + and , (since +

the number of matrices of the first form is a:# "b: :$ :. For the second form, there are no restrictions
and , can be anything from J , except for both !). Once + and , are fixed, there are : choices for - . Hence,

on . or /, so the number of choices is :# . Hence, in total, the number of non-invertible matrices in KP# a: b
is :$ : :# :$ :# :. Finally, a matrix is either invertible or not, so if Q# aJ b is the set of # #
matrices with entries in J ,
kKP# a: bk kQ# a: bk keE Q# a: b E not invertiblefk :% a:$ :# :b :% :$ :# :. 

Problem 1.4.8 Show that KP8 aJ b is non-abelian for any 8 # and any J .

Assume E a+34 b F a,34 b KP8 aJ b. Then the top-left value for EF would be !85" +"5 ,5" and the top-
Proof. A simple but not rigorous way of showing this is to consider just the outer edges of a matrix.

left value for FE would be !85" +5" ,"5 . These do not have to be equal, although this is not immediately
obvious, since the condition that E and F have an inverse could place an important restriction on +34 and ,34
so that these two sums are indeed equal.

To see a more specific example of this, let ! "aJ b and " "aJ b . That is, let ! and " denote the

Consider the matrix given by E a+34 b such that +34 $3a84"b , where $ is the Kronecker delta (i.e., let E
additive and multiplicative identity in J , respectively. Finally, let " denote the additive inverse of ".
Robert Krzyzanowski Solutions to Dummit and Foote Chapter 1

be the matrix with " in the bottom-left to top-right diagonal); and consider the matrix F a,34 b with
,34 " $34 except for ,88 " (i.e., F is the matrix with "'s everywhere except the main diagonal, with the

similarly det F a"b8+1 (the zero's in the diagonal mean the determinant is just the appropriate sign times
additional exception that the bottom-right element is "). Then notice that det E " (depending on 8) and

the bottom-right element, "). Hence E F KP8 aJ b. Finally, it's then simple to show that EF is F mirrored

left corner whereas the latter does not), so KP8 aJ b is not abelian. 
vertically and FE is F mirrored horizontally. These are different matrices (the former has a " in the lower

Problem 1.4.9 Prove that the binary operation of matrix multiplication of # # matrices with real number
entries is associative.

We can prove this in general for 8 " and elements in any field J . Let E a+34 b F a,34 b
G a-34 b KP8 ab with +34 ,34 -34 J for 3 4 e" 8f. Then component-wise:
Proof.

aEF bG aa+34 ba,34 bba-34 b ! +35 ,54 a-34 b ! ! +35 ,52 -24 , and
8 8 8

5" 2" 5"

EaFG b a+34 baa,34 ba-34 bb a+34 b ! ,32 -24 ! +35 ! ,52 -24 .
8 8 8

2" 5" 2"

Notice these last two are equivalent, since we can rearrange terms due to the abelian nature of addition and
multiplication in J (this is what gives associativity). 

Problem 1.4.10 Let K


-
l + , - + ! - ! .
+ ,
!

(a) Compute the product of


-"
and #
-#
+" ," + ,#
to show that K is closed under matrix multiplication.
! !

(b) Find the matrix inverse of


-
+ ,
and deduce that K is closed under inverses,
!
(c) Deduce that K is a subgroup of KP# ab.

Solution.

! -" ! -# ! K.
+" ," +# ,# +" +# +" ,# ," -#
(a)
-" -#

! - ! "- ! "
+ , "+ ,+- " !
(b) .

Notice the inverse is in K (since + ! - ! means all the entries are real) so K is closed under inverses.
(c) From Problem 1.4.9, K is associative since it is a subset of # # matrices with real number entries.
Additionally, "! !" is an identity for K, so that all the group axioms are satisfied. Then K is a group, and it is a
subset of KP# ab, so it must be a subgroup. 
The next exercise introuces the Heisenberg group over the field J and develops some of its basic properties.
When J this group plays an important role in quantum mechanics and signal theory by giving a group
theoretic interepretation (due to H. Weyl) of Heisenberg's Uncertainty Principle. Note also that the
Heisenberg group may be defined more generally -- for example, with entries in .
Robert Krzyzanowski Solutions to Dummit and Foote Chapter 1


" + ,
Let L ! " - l + , - J

! ! "
Problem 1.4.11 -- called the Heisenberg group over J . Let

" + , " . /
" - and ] ! " 0 be elements of L aJ b.
! ! " ! ! "
\ !

(a) Compute the matrix product \] and deduce that L aJ b is closed under matrix multiplication.
Exhibit explicit matrices such that \] ] \ (so that L aJ b is always non-abelian).
(b) Find an explicit formula for the matrix inverse \ " and deduce that L aJ b is closed under inverses.
(c) Prove the associative law for L aJ b and deduce that L aJ b is a group of order kJ k$ .
(d) Find the order of each element of the finite group L a#b.
(e) Prove that every nonidentity element of the group Lab has infinite order.

Solution. (a)

" + , " . / " +. / +0 ,

! " !
\] ! " - ! " 0 ! " -0 .
! " ! ! ! "

Since J is closed under and , + . / +0 , - 0 J so that \] L .

(b)

" + , " ! ! " ! , +- " + ! " ! ! " + +- ,


-! - ! ! ! " !!
! " ! " ! ! " !
! " " ! ! " " " - .
! " ! ! ! " ! ! ! "

Hence,

" + +- ,

!
"
\ ! " - K
! "

since + +- , - J . Hence, L aJ b is closed under inverses.

(c) This follows Problem 1.4.9. However, it is possible to do it explicitly:

" + , " . / " 1 2 " +. / +0 , " 1 2

! " ! "
! " - ! " 0 ! " 3 ! " -0 ! " 3
! " ! ! " ! ! ! " ! !

" a+ . b 1 2 a+ . b3 a/ +0 ,b " + a. 1b a2 .3 /b +a0 3b ,


a- 0 b 3 - a0 3b
! !
! " ! "
! " ! "

" + , " .1 2 .3 / " + , " . / " 1 2

! !
! " - ! " 0 3 ! " - ! " 0 ! " 3 .
! " ! ! " ! " ! ! " ! ! "
Robert Krzyzanowski Solutions to Dummit and Foote Chapter 1

(d) First, notice

" #+ +- #, " ! +-

! ! "
E# ! " #- ! " ! , and
! " !

" %+ '+- %,

!
%
E ! " %- ".
! "

Hence each non-identity element has order between # and %. If +- ! (6 possible matrices), the matrix has
order #. Otherwise, it's easily checked the cases + - ", , ! or " yield a matrix of order %.

(e) We will show inductively that


8a8"b
" 8+ # +- 8,

!
8
E ! " 8- .
! "

This is trivial for 8 ". Assume E5 has this form for some 5 ". Then
5 a5"b
" 5+ # 5, "
+- + ,

! "
5"
E E5 E ! " 5- ! " -
! " ! !
" , 5+- 5 a5" b
+- 5, " a5 "b+ 5 a5"b
a5 "b,
a 5 "b -
5+ + # # +-

! !
! " 5- - ! " .
! " ! "
Robert Krzyzanowski Solutions to Dummit and Foote Chapter 1

Chapter 1.5
Problem 1.5.1 Compute the order of each of the elements in U) .

Solution. The orders of " and " are " and #, respectively, and everything else has order %. 

Problem 1.5.2 Write out the group tables for W$ H) and U) .

Solution.
Group table for W$ :
aW$ b " a" #b a" $ b a# $ b a" # $b a " $ #b
a " #b a" $ b a# $ b a" # $b a" $ #b
a " #b a" # b a" # $ b a" $ #b a" $b a# $b
" "

a " $b a" $ b a" $ # b a" # $b a# $b a" #b


"

a # $b a# $ b a" $ # b a" $ #b a" #b a " $b


"

a " # $b a" # $ b a# $ b a" #b a" $b a" $ #b


"

a " $ #b a" $ # b a" $ b a# $b a" #b a" # $b


"
"
Group table for H) :
aH ) b " < <# <$ = =< =<# =<$
" " < <# <$ = =< =<# =<$
< < <# <$ " =<$ = <= <=#
<# <# <$ " < =<# =<$ = <=
<$ <$ " < <# =< =<# =<$ =
= = =< =<# =<$ " < <# <$
=< =< =<# =<$ = <$ " < <#
=<# =<# =<$ " =< <# <$ " <
=<$ =<$ = =< =<# < <# <$ "
Group table for U) :
aU) b " " 3 3 4 4 5 5
" " " 3 3 4 4 5 5
" " " 3 3 4 4 5 5
3 3 3 " " 5 5 4 4-*
3 3 3 " " 5 5 4 4
4 4 4 5 5 " " 3 3
4 4 4 5 5 " " 3 3
5 5 5 4 4 3 3 " "
5 5 5 4 4 3 3 " "

Problem 1.5.3 Find a set of generators and relations for U)

Proof. Notice we need two generators. Take B C. We will show necessary relations are
B# C# and CBC" B" .
Robert Krzyzanowski Solutions to Dummit and Foote Chapter 1

If B 3 C 4, and BC 5 , it's easy to see U) is generated by and satisfies these relations. Namely, " 3% ,
" 3# , 3 3$ , 4 4$ , 5 a43b$ and 5 a34b$ . A group of order ) satisfying this presentation exists,
so it must be of order 8 at least. [Stop reading here. Need to finish this.]
Notice " 3 3# 3$ must all be different elements. Additionally, from the third relation a34b$ 4$ 3$ so that
a43b% ", and also a34b# 34 4$ 3$ so that a34b# a34b4# 3# a34b. But this gives 4# 3# ", and from the first two
relations 3# 4# " and 3# 4# . Additionally, 4# 3# " gives us how to commute 3 and 4, namel
Hence, all powers of 4 greater than " can be expressed in 3, so only 4 itself can be unique. To verify it is,
we see 4 3< for some ! < % gives a contradiction: if < were odd the fact 3# 4# " would give 35 " for
a " 5 %, a contradiction; if < were even, 3# 4# " would then give a contradiction as well. Again,
43 3< , 43# 3< or 43$ 3< would give a similar contradiction, so that 4 43 43# , and 43$ are all distinct
elements. Adding any further 3 or 4's to " 3 3# 3$ 4 43 43# or 43$ will give use a reduction using our
relations, so these are the only possible elements. Hence,
U) 3 4 l 3% 4% a34b% ".
Robert Krzyzanowski Solutions to Dummit and Foote Chapter 1

Chapter 1.6
Problem 1.6.1 Let : K L be a homomorphism.
(a) Prove that :aB8 b :aBb8 for all 8 .
(b) Do part (a) for 8 " and deduce that :aB8 b :aBb8 for all 8 .

Proof. (a) This is a simple inductive argument. Assume :B5 :aBb5 for some 5 . Then
:B5" :B 5 B :B5 :aBb :aBb5 :aBb :aBb5" .
(b) First, notice :aB! b :a"b :aBb! ". This is because :a"b :a" "b :a"b:a"b. Furthermore,
" :a"b :aB" Bb :aB" b:aBb :aBB" b :aBb:aB" b, so that :aB" b :aBb" . Hence, for
8 ,
:aB8 b :aB" b :aB" b :aBb" :aBb8 ,
8 8 8

so that the statement holds for all 8 . 

Problem 1.6.2 If : K L is an isomorphism, prove that k:aBbk kBk for all B K. Deduce that any two
isomorphic groups have the same number of elements of order 8 for each 8 . Is the result true if : is
only assumed to be a homomorphism?

Proof. Let k:aBbk 8 so that :aBb8 ". Of course :a"b " since
:a"b :a" "b :a"b:a"b
so by cancellation :a"b ". Then :aBb8 :a"b. However, :aBb8 :aB8 b from the previous exercise.
Hence, :aB8 b :a"b and since isomorphisms are injective, B8 ". Assume kBk was smaller, say 7 8.
But then :aB7 b " so that :aBb7 ", giving k:aBbk 7 8, a contradiction. Hence k:aBbk kBk. Let
K8 and L8 be the subsets of K and L with order 8. Since : is an isomorphism, :aBb L8 is unique for
each B K8 . Hence, kK8 k kL8 k, that is, two isomorphic groups have the same number of elements of order
8 for each 8 . This result is not true if : is a homomorphism. For example, if : is prime then
: : : :, + , +
is a homomorphism such that : : has :# " elements of order : whereas : only has : ".

Problem 1.6.3 If : K L is an isomorphism, prove that K is abelian if and only if L is abelian. If


: K L is a homomorphism, what additional conditions on : (if any) are sufficient to ensure that if K is
abelian, then so is L ?

Assume K is abelian. If + , L , there are unique B C K such that + :aBb and


C :a,b. Then +, :aBb:aCb :aBCb :aCBb :aC b:aBb ,+. Assume L is abelian. If B C K,
Proof.

then :aBCb :aBb:aC b :aC b:aBb :aCBb so that BC CB by the fact : is injective. Hence K is abelian
if and only if L is. If : is a homomorphism and K is abelian, then for B C K, :aBb:aCb :aBC b
:aCBb :aCb:aBb. Hence, for two elements + , L , if there are B C K such that + :aBb and
, :aCb, then we can say the two elements commute. In other words, : has to be surjective (but not
necessarily injective). 

Problem 1.6.4 Prove that multiplicative groups e!f and e!f are not isomorphic.
Robert Krzyzanowski Solutions to Dummit and Foote Chapter 1

The group e!f has no elements of order %, but e!f has two ( 3), so by Problem
1.6.2 the groups are not isomorphic. To verify the former rigorously, notice if B e!f with finite order
Proof.

5 , then B5 " and kBk5 B5 " (where k k denotes absolute value, not order). But then kBk " (since the
only non-negative real 5 -th root of unity is "), so that either B " which has order ", or B " which has
order #. 

Problem 1.6.5 Prove that the additive groups and are not isomorphic.

Proof. There is no injection from to since the former is uncountable and the latter is countable.

if B , then ! :a!b :aB Bb :aBb :aBb so that :aBb and :aBb are inverses in a b.
This can be seen more explicitly in the following argument. Let : be an isomorphism. First, notice

Assume :a"b +, . From the previous result, we can consider + , ! without loss of generality. Then
:a,b ,:a"b +. Thus, :a,b :+ +, + : +, + ". Then : +, " (otherwise we would get
+ - + for some - " in a b). Let .- be a lowest form representation. Then
: -, , -, .,- ,- ,- -
for some B .- in a b). From earlier, this implies : +.,-
+ - : + - , so that - : + : +. . : +. . This implies : +. . (otherwise, .B -
.- . This applies for any rational in ,
so that : must map all rationals to rationals (: l ). However, this contradicts the fact : is injective,
since any irrational in must also map to a rational. 

Problem 1.6.6 Prove that the additive groups and are not isomorphic.

Let : be an isomorphism. First, notice if B , then ! :aB Bb


:aBb :aBb so that :aBb and :aBb are inverses in a b. Assume :a"b +, . Then for 5 ,
Proof.

:a5 b :a5 "b 5:a"b +5 , . From earlier, :a5 b , . Hence, all elements in must be mapped to
+5

integer multiples of +, . However, then no element will be mapped to, e.g., #,


+
. This contradicts the fact : is
surjective. 

Problem 1.6.7 Prove that H) and U) are not isomorphic.

Proof. The group H) only has two elements of order % (< and <$ ), whereas U) has six. By Problem
1.6.2, they cannot be isomorphic. 

Problem 1.6.8 Prove that if 8 7, W8 and W7 are not isomorphic.

Proof. This is obvious, since then 8x 7x so that kW8 k kW7 k, and there is no bijection between
finite sets of different cardinality. 

Problem 1.6.9 Prove that H#% and W% are not isomorphic.

Proof. From Problem 1.3.4, we know W% has six elements of order #, whereas H#% has "$ (=<3 for
! 3 "#, and =' ). By Problem 1.6.2, they cannot be isomorphic. Alternatively, notice by Problem 1.3.15
that the only possible orders of elements of W% are in e" # $ % lcma# #bf e" # $ %f. However, H#% has
an element of order "# (namely <). 

k?k kHk as follows: let ) ? H be a bijection. Define


Problem 1.6.10 Fill in the details of the proof that the symmetric groups W? and WH are isomorphic if

: W? WH by :a5b ) 5 )" for all 5 W?


and prove the following
Robert Krzyzanowski Solutions to Dummit and Foote Chapter 1

(a) : is well-defined, that is, if 5 is a permutation of ? then ) 5 )" is a permutation of H.


(b) : is a bijection from W? onto WH . [Find a 2-sided inverse for :.]
(c) : is a homomorphism, that is, :a5 7 b :a5 b :a7 b.
Note the similarity to the change of basis or similarity transformations for matrices.

Proof. (a) Notice ) ? H, 5 ? ?, and )" H ?, so that


) 5 )" H H.
This is a definition of ) 5 )" being a permutation of H, so : is indeed well-defined.
(b) A two-sided inverse is :" ) 5" )" :
: :" ) 5 )" ) 5 " )" ) 5 5 " )" ) )" ", and
:" : ) 5" )" ) 5 )" ) 5 " 5 )" ) )" ".
(c) :a5 7 b ) 5 7 )" ) 5 )" ) 7 )" :a5 b:a7 b. 

Problem 1.6.11 Let E and F be groups. Prove that E F z F E.

Let a+ ,b a, +b. If :a+ ,b :a- . b (with + - E and , . F) then :a+ ,b a, +b


:
and :a- . b a. - b so that a, +b a. - b. But then , . and + - , so that a+ ,b a- .b. Assume
Proof.

a, +b F E. Then :a+ ,b a, +b so that :" exists for all elements in F E. Hence, : is a bijection.
Finally, if a+ ,b a- . b E F , then (leaving group operations implicit)
:aa+ ,ba- . bb :a+- ,. b a,. +- b a, +ba. - b :a+ ,b:a- .b. 

Problem 1.6.12 Let E, F , and G be groups and let K E F and L F G . Prove that K G is
isomorphic to E L .

In other words, prove aE F b G z E aF G b. Let : aE F b G E aF G b


be defined by :aa+ ,b - b a+ a, - bb. If :aa+ ,b - b :aa. /b 0 b, then a+ a, - bb a. a/ 0 bb so that
Proof.

+ . and a, - b a/ 0 b and hence , / and - 0 . But then aa+ ,b - b aa. /b 0 b. Now take
a+ a, - bb E L . Then :aa+ ,b - b a+ a, - bb so that :" exists for all elements in E L . Finally, for
+ . E , / F and - 0 G (leaving group operations implicit)
:aaa+ ,b - baa. /b 0 bb :aa+ ,ba. /b -0 b :aa+. ,/b -0 b a+. a,/ -0 bb
a+. a, - ba/ 0 bb a+ a, - bba. a/ 0 bb :aa+ ,b - b:aa. /b 0 b. 

:aKb, is a subgroup of L . Prove that if : is injective then K z :aKb.


Problem 1.6.13 Let K and L be groups and let : K L be a homomorphism. Prove that the image of :,

Let B C :aKb L with + , K such that :a+b B and :a,b C. Then :a+b:a,b
:a+,b, and since +, K, :a+,b :aKb. Similarly, :a,b:a+b :aKb. Hence, the operation is closed in
Proof.

:aKb. Furthermore, for B :aKb, let C K be such that :aCb B. Then :aC" b :aC b" B" :aKb,
so that :aKb is closed under inverses. Since L is a group and :aKb L , associativity holds. Hence :aKb is
a subgroup of L . If : is injective, then since :aKb e2 L l b1 K s.t. :a1b 2f, by definition if
2 :aKb, then there is 1 K such that :a1b 2. Hence, : is injective. This gives a bijective
homomorphism from K to :aKb, so that K z :aKb. 
Robert Krzyzanowski Solutions to Dummit and Foote Chapter 1

be e1 K l :a1b "L f. Prove that the kernel of : is a subgroup of K. Prove that : is injective if and only if
Problem 1.6.14 Let K and L be groups and let : K L be a homomorphism. Define the kernel of : to

the kernel of : is the identity subgroup of K.

If B C ker :, then :aBCb :aBb:aCb "L "L "L so that BC ker :. Hence it is closed
under the operation. Next, notice "L :a"K b :aBB" b :aBb:aB" b "L :aB" b :aB" b so that
Proof.

B" ker :. Hence it is also closed under inverses. Since "K ker : and associativity follows from the fact
K is a group, ker : is itself a group and hence a subgroup of K. If : is injective, then there can only be one

:a1b :a2b, 1 2 would give a contradiction, as then "L :a1b:a2b" :a12" b so that 12" "K is
element mapped to "L . This must necessarily be "K , so that ker : " K. Conversely, if ker : ", then if

in ker :. 

Problem 1.6.15 Define a map 1 # by 1aaB Cbb B. Prove that 1 is a homomorphism and find the
kernel of 1.

Proof. We easily see that

1aaB Cb aBw Cw bb 1aaB Bw C Cw bb B Bw 1aaB C bb 1aaBw Cw bb.


As for the kernel,
1aaB Cbb ! B ! ker 1 ea! Cb l C f. 

1" K E and 1# K F defined by 1" aa+ ,bb + and 1# aa+ ,bb , are homomorphisms and find
Problem 1.6.16 Let E and F be groups and let K be their direct product, E F . Prove that the maps

their kernels.

Proof. We easily see that


1" aa+ ,ba+w ,w bb 1" aa++w ,,w bb ++w 1" aa+ ,bb1" aa+w ,w bb, and
1# aa+ ,ba+w ,w bb 1# aa++w ,,w bb ,,w 1# aa+ ,bb1# aa+w ,w bb.
Additionally, it's clear that
1" aa+ ,bb ! + ! ker 1" ea! ,b l , F f, and
1# aa+ ,bb ! , ! ker 1# ea+ !b l + F f.
In general, 15 #3M E3 E5 with a+" b +5 (for E3 groups) is a homomorphism with kernel
! ! +5 ! l +5 E5 . 
5 th term
Problem 1.6.17 Let K be any group. Prove that the map from K to itself defined by 1 1" is a
homomorphism if and only if K is abelian.

Call the map :. Then for + , K, :a+,b a+,b" ," +" . This equals +" ,"
:a+b:a,b if and only if ," +" +" ," if and only if ,+ +, (multiply by +, on the right and ,+ on the
Proof.

left). 

Problem 1.6.18 Let K be any group. Prove that the map from K to itself defined by 1 1# is a
homomorphism if and only if K is abelian.
Robert Krzyzanowski Solutions to Dummit and Foote Chapter 1

Proof. Call the map :. Then for + , K, :a+,b a+,b# +,+,. This equals +# ,# :a+b:a,b if
and only if +,+, +# ,# if and only if ,+ +, (cancel the +'s on the left and ,'s on the right). 

Problem 1.6.19 Let K eD l D 8 " for some 8 f. Prove that for any fixed integer 5 " the map
from K to itself defined by D D 5 is a surjective homomorphism but not an isomorphism.

Call the map :. First, notice :aD" D# b aD" D# b5 D"5 D#5 :aD" b:aD# b. To see it is surjective,
let D . By definition, D 8 " for some 8 so thus " aD 8 b55 D "5 so that D "5 K with
Proof.
85

:D "5 D . The map is not isomorphic because if D " is a 5 -th root of unity (from complex analysis we
know such a number exists for 5 "), then D 5 " and "5 ". 

Problem 1.6.20 Let K be a group and let AutaKb be the set of all isomorphisms from K onto K. Prove that
AutaKb is a group under function composition (called the automorphism group of K and the elements of
AutaKb are called automorphisms of K).

Sketch. Let : < AutaKb. Then : < K K, and since composition of bijective functions is
bijective, : < AutaKb so that the set is closed. Furthermore, " : : " :, and the inverse function
:" is an isomorphism with the property :" : : :" ". Finally, associativity follows from
associativity of functional composition. 

Proof. Let : < AutaKb. Then : < K K. Furthermore, for B C K,


a: <baBC b :a<aBCbb :a<aBb<aC bb :a< aBbb :a< aC bb a: < baBb a: < baC b,
so that : < is a homomorphism. Assume a: <baBb a: < baC b, that is, :a<aBbb :a<aCbb. Injectivity
of : implies <aBb <aC b, and injectivity of < implies B C. Therefore, : < is injective. Furthermore, if
D K, then by surjectivity of :, there is a C K such that :aCb D . By surjectivity of < , there is an B K
such that <aBb C, so that D :aCb :a<aBbb a: < baBb. Hence, : < is surjective, and so indeed an
isomorphism and thus in AutaKb. This proves closure.
To show there is an identity, consider " K K given by 1 1. Then for B K,
a: "baBb :a"aBbb :aBb and a" :baBb "a:aBbb :aBb.
By definition " is the identity in AutaKb. Since : is a bijection, it has an inverse function :" such that
a: :" baBb a:" :baBb B for all B K. This is the definition of :" being the inverse of : in
AutaKb. Finally, composition of functions is associative in general, and for : < 9 AutaKb with B K,
aa: <b 9baBb a: < ba9aBbb :a< a9aBbbb :aa< 9baBbb a9 a< 9bbaBb.
This means that AutaKb satisfies all the group axioms so that it is indeed a group under function
composition. 

Problem 1.6.21 Prove that for each fixed nonzero 5 the map from to itself defined by ; 5; is an
automorphism of .

Call the map :. For : ; , :a: ; b 5 a: ; b 5: 5; :a:b :a; b. To show


bijectivity, notice :a:b :a; b means 5: 5; so that division gives : ; ; if ; , then ;5 gives
Proof.

:a;5 b 5 a;5 b ; so that : is surjective. 

Problem 1.6.22 Let E be an abelian group and fix some 5 . Prove that the map + +5 is a
homomorphism from E to itself. If 5 ", prove that this homomorphism is an isomorphism.
Robert Krzyzanowski Solutions to Dummit and Foote Chapter 1

Proof. If + , E, then a+,b5 +5 ,5 since E is abelian (Problem 1.1.24). If 5 ", then if


+" ," , then " ++" +," ,," so that cancellation on the right by ," gives + ,. To show
surjectivity, notice if + E, then of course a+" b +. 
"

Problem 1.6.23 Let K be a finite group which possesses an automorphism 5 such that 5a1b 1 if and only
if 1 ". If 5# is the identity map from K to K, prove that K is abelian (such an automorphism 5 is called
fixed point free of order #).

We claim there is a bijection B B" 5aBb. To show this map is injective, let B" 5aBb
C" 5aCb. Then 5 aBb BC" 5 aC b so that 5 aBb5 aC b" 5 aBC" b BC" . Since 5 is fixed point free,
Proof.

BC" ", so that C B. Since this is a finite map from K K and it is injective, it must be surjective. In
other words, each element 1 K can be written 1 B" 5aBb for some B K.
To show that K is abelian, notice if 1 B" 5aBb for any 1 K, then
5a1b 5 aB" 5aBbb 5 aBb" B aB" 5 aBbb
"
1" ,
and apply Problem 16.17 (which states that a group possessing the homomorphism 1 1" is abelian). 

Prove that K z H#8 , where 8 kBCk.


Problem 1.6.24 Let K be a finite group and let B and C be distinct elements of order # in K that generate K.

Proof. Let > BC. Then from Problem 1.2.6, these elements give a presentation

> B l >8 B# ", BC >B B>" .

this explicitly. Let : H#8 K given by :a=3 <4 b B3 >4 . Then B and > in K satisfy all the relations that =
Notice this is the standard presentation of H#8 . This is enough to guarantee isomorphism, but we can show

and < do in H#8 . Hence, if :a=3 <4 b :a=8 <7 b with 3 8 e! "f 4 7 e! 8 "f, then B3 >4 B8 >7 .
Assume 3 8. Without loss of generality, let 3 ! and 8 ". Then >4 B>7 , so that B >47 , which is a
contradiction, since B and > satisfy the same relations in K and = and < do in H#8 , specifically = <3 for any 3
(respectively, B >3 for any 3, e.g., 3 4 7). Hence, 3 8. But then either way (by multipling B on the

amod 8b, which induces B>4 B>7 for 4 7 amod 8b. Since 4 7 e! 8 "f, we have 4 7. Hence,
left if there is no B), this means B>4 B>7 . However, again, we know H#8 has the relation =<3 =<4 for 3 4

know every element can be written this way since K has the same relations as H#8 ), then :a=3 <4 b B3 >4 .
3 8 and 4 7, so that =3 <4 =8 <7 . This shows injectivity. Surjectivity is trivial, since if B3 >4 K (and we

Finally, homomorphism follows from:


:a=3 <4 =8 <7 b :a=3 =8 <4 <7 b :a=38 <74 b B38 >74 B3 B8 >4 >7 B3 >4 B8 >7 :a=3 <4 b:a=8 <7 b,
where the penultimate equality follows again from the fact B and > obey the same relations in K as = and < do
in H#8 . 

Problem 1.6.25 Let 8 , let < and = be the usual generators of H#8 and let ) #18.

(a) Prove that the matrix


sin ) cos )
cos ) sin )
is the matrix of the linear transformation which rotates the
B, C plane about the origin in a counterclockwise direction by ) radians.
(b) Prove that the map : H#8 KP# ab defined on generators by

: a< b and :a=b


cos ) !
cos ) sin ) ! "
sin ) "
Robert Krzyzanowski Solutions to Dummit and Foote Chapter 1

extends to a homomorphism of H#8 into KP# ab.


(c) Prove that the homomorphism : in part (b) is injective.

Proof. (a) Let aB Cb # . Using polar coordinates, this point is a< cos < < sin <b. Rotating by )
means the new point is (represented as a vector)
< cos a< ) b < acos < cos ) sin < sin ) b
< sin a< ) b < acos < sin ) sin < cos ) b B sin ) C cos ) sin ) cos ) C
B cos ) C sin ) cos ) sin ) B
.

(b) Notice that from the previous exercise, the matrix < is mapped to rotates the B C plane by ) radians, so
that applying it : times leads to a rotation by :) radians, that is,

sin ) cos )

cos :)
:
cos ) sin ) cos :) sin :)
sin :)

evidence. Then if we let =a4b 4 if 8 ! and 4 if 8 ",


for all : . This can be proved using a simple inductive argument, but we will take our intuition as

:a=3 <4 =8 <7 b :=38 <=a4b7


! sin ) cos )
38 =47
! " cos ) sin )
"

cos ) sin ) cos )


3 8 =4 7
! " ! " cos ) sin ) cos ) sin )
" ! " ! sin )

cos ) " cos )


3 =4 8 7
! " cos ) sin ) ! " cos ) sin )
" ! sin ) ! sin )
:a=3 <4 b:a=87 b,
where the middle step is trivial if 8 ! (so that =a4b 4), and for 8 ",

" ! sin ) cos )



! sin ) cos )
8 =4 4
! " cos ) sin ) ! " cos ) sin )
"


! sin -4) cos -4) " ! sin 4) cos 4)
! " cos -4) sin -4) ! " cos 4) sin 4)

"


sin 4) sin 4) cos 4) " ! sin ) cos ) " !
=4 8
sin 4) cos 4) cos 4) sin 4) ! " cos ) sin ) ! "
.
cos 4)
(c) Assume

:a= < b :a= < b, or


! sin ) cos )

! sin ) cos )
3 4 8 7
3 4 8 7 ! " cos ) sin ) ! " cos ) sin )
" "
where 3 8 e! "f and 4 7 e! 8 "f. However, from the initial remark in the solution of part (b),

" ! sin 4) cos 4) " ! sin 7) cos 7)


3 8
! " cos 4) sin 4) ! " cos 7) sin 7)
(1)

Furthermormore, notice
Robert Krzyzanowski Solutions to Dummit and Foote Chapter 1

"
sin :)
! " cos :) sin :) sin :) cos :)
.
! sin :) cos :) cos :)
Hence, if 3 8 and assuming without loss of generality that 3 " so that 8 !, we would need



cos 7) sin 4)

cos 4) sin 7)

cos 4) sin 7)
(2)
cos 7) sin 4)

in order for the two matrices in (1) to be equal. However, 4 7 e! 8 "f, so ! 4) #1 and
! 7) #1. Notice the first and last equation in (2) force 4) e! 1f so 7) 1# $#1 , and the second
and third equation give 7) e! 1f and so 4) 1# $#1 . Clearly, these four equations have no solution, so
that 3 8. We may multiply by the inverse of !" "! if 3 8 ". Then

sin 4)
cos 7)
cos 4) sin 4) cos 7) sin 7)
cos 4) sin 7)
so that we have the equations cos 4) cos 7) and sin 4) sin 7). This means 4) 7) amod #1b, but
since 0 4) #1 and ! 7) #1, this means 4) 7) and so 4 7. Hence, 3 8 and 4 7 so that
=3 <4 =8 <7 and the map : is injective. 

KP# ab defined on generators by


Problem 1.6.26 Let 3 and 4 be generators of U) described as in Section 5. Prove that the map : from U) to

-"
:a3b and :a4b "
!
! ! "
! -"

extends to a homomorphism. Prove that : is injective.

Proof. First, we need to know the values of the ::

:a"b :a4 b :a4b:a4b


!

"
#
! " " !
#
" "KP# ab , and
" !
-" -"
:a5 b :a34b :a3b:a4b "
! -" ,
! ! " !

! -" !
where :a3b :a4b :a5 b can easily be seen to be :a3b :a4b and :a5 b seen as multiplying the
scalar " by each matrix (the action of :a"b). Then :aB "b :aBb:a "b :a " Bb for each
B e3 4 5 f. We already know :a34b :a3b:a4b. For all the others,
-" -" -"
:a43b :a5 b " :a4b:a3b,
-" -"
! ! ! " !
! -" ! ! !

arcsin " " 1 1 gives 4) 7). Then take 4) and 7) amod #1b, so that 4 7. Alternatively, we could have noticed
There also more fun ways of showing this. For example, if we let 4) 7) 1 1 for a moment, then the injectivity of

. ) asin 4)b . ) asin 7)b so that 4 cos 4) 7 cos 7), but since cos 4) cos 7) 4 cos 4) 7 cos 4) so that 4 7.
. .
Robert Krzyzanowski Solutions to Dummit and Foote Chapter 1

-" -"
:a53b :a4b :a5 b:a3b,
! -"
! " ! !

" ! ! -"
-" -"
:a35 b :a4b :a3b:a5 b,
! "

! " ! " ! !

" ! ! -" -" !
-" -"
:a45 b :a3b :a4b:a5 b, and
-" -" !
! ! " !
! " !

-" -"
:a54b :a3b :a3b " :a5 b:a4b.
!

-" -"
! ! ! "
! !

We can look at the matrices e:aBb B U) f im : and see that there are no two same :aBb :aC b for
B C, so that : is injective by inspection. 
Robert Krzyzanowski Solutions to Dummit and Foote Chapter 1

Chapter 1.7
Problem 1.7.1 Let J be a field. Show that the multiplicative group of nonzero elements of J (denoted by
J ) acts on the set J by 1 + 1+, where 1 J , + J and 1+ is the usual product in J of the two field
elements (state clearly which axioms in the definition of a field are used ).

Proof. Let 1" 1# J and + J (as a set). Then 1# + 1# + J where 1# + is the product in J as

elements, and 1# J implies 1# !. Furthermore, then 1" a1# +b 1" a1# +b, where 1# + is taken as in J
a field. Notice this is possible since multiplication in J as an abelian group is only defined for non-zero

and 1" is taken as in the group J . Since multiplication in the field J is a group, and similarly 1" !, we
can say 1" a1# +b a1" 1# b+ by group associativity. Hence, 1" a1# +b a1" 1# b +. Finally, if + J , then for
" J , " + "a+b +, since " is the multiplicative identity in the field J . This shows J satisfies the
definition of being an action on J . 

Problem 1.7.2 Show that the additive group acts on itself by D + D + for all D + .

as the group operation in a b. We then get


Proof. Let D" D# as a group and + as a set. Then D# + D# + where the sum is computed

D" aD# +b D" aD# +b aD" D# b + aD" D# b +

explicitly when writing aD" D# b +. Finally, " + ! + +. 


since addition in as a group is associative. Notice since the group operation is addition here, we showed it

Note. These first two problems were trivial, but showed the care that must be taken when considering the

group operation of a b applied to the elements D + a b). From now on, we will drop such
operations being applied (e.g., even though D + was an element of the set , the actual element was the

pedantry and immediately become less formal by treating the operations implicitly (unless some extreme
trickery is at hand!).

Problem 1.7.3 Show that the additive group acts on the B C plane ,C < aB Cb aB <C Cb.

Proof. Let < = and aB Cb . Then


< a= aB Cbb < aB =C Cb aaB =Cb <C Cb aB a< =bC Cb a< =b aB C b,
and "ab aB Cb aB !C Cb aB Cb. 

Problem 1.7.4 Let K be a group acting on a set E and fix some + E Show that the following sets are
subgroups of K.
(a) the kernel of the action,
(b) e1 K l 1+ +f this subgroup is called the stabilizer of + in K.

First, notice that 1" 1# is also in the kernel of the action, since a1" 1# b + 1" a1# +b 1" + +. Thus, the
Proof. (a) Let 1 1" 1# be in the kernel of the action. Then 1 + 1" + 1# + + for all + E.

kernel is closed under the group operation. Additionally, 1" + 1" a1 +b a1" 1b + " + +, so
that the kernel is closed under inverses. Hence, it is a subgroup of K.

shows e1 K l 1+ +f is a subgroup of K. 
(b) Replace "kernel [of the action]" by this set and remove "for all + E" from the proof of part (a) and this
Robert Krzyzanowski Solutions to Dummit and Foote Chapter 1

Problem 1.7.5 Prove that the kernel of an action of the group K on the set E is the same as the kernel of the
corresponding permutation representation K WE .

Proof. Let 1 be in the kernel of the action. Then 1+ + a+ E so the permutation representation of

permutation representations, :a1b "WE . But this is the definition of 1 being in the kernel of the permutation
1 is the trivial permutation " WE . That is, if we let : K WE be the map that takes elements in K to their

representation (see Problem 1.6.14). Conversely, let 1 K such that :a1b "WE . Then :a1ba+b +.
However, by definition :a1ba+b 1 +, which means 1 is in the kernel of the action. 

Problem 1.7.6 Prove that a group K acts faithfully on a set E if and only if the kernel of the action is the set
consisting only of the identity.

Proof. Assume K acts faithfully on E, that is, permutation representations are unique. Notice that for

K such that 1 + +. But this means the kernel of the action is e"f. Now assume the converse. By way of
" K (the identity), by definition " + + a+ E. Since K acts faithfully, this must be the only element in

contradiction, assume 1" + 1# + +w for all + E for different 1" and 1# in K. But then

" + 1" a1" +b a1" 1" b+ a"b+ + and 1# + 1# a1# +b a1# 1# b+ a"b+ +.
1" w " " " w " "

" + 1# + +. However, since e1" + + Ef e1# + + Ef is a permutation on E, this


That is, 1" w " w

means that e+w 1" + 1# + + Ef is also a (indeed the same) permutation. Hence, 1" w " w
" + 1# + + for
all +w E. However, if 1" 1# , then 1" " " " "

then 1" + + for all + E gives a contradiction, since we assumed the kernel of the action is e"f, and
" 1# so that one of 1" and 1# is not the identity, say 1" . But
" w w

1" "
" " so that 1" is not in the kernel. 

Problem 1.7.7 Prove that in Example 2 in this section the action is faithful.

Proof. For completeness, Example 2 is copied here below:


Example 2. The axioms for a vector space Z over a field J include the two axioms that the multiplicative
group J act on the set Z . Thus vector spaces are familiar examples of actions of multiplicative groups of
fields where there is even more structure (in particular, Z must be an abelian group) which can be exploited.
In the special case when Z 8 and J the action is specified by
!a<" <8 b a!<" !<8 b
for all ! , a<" <8 b 8 , where !<3 is just multiplication of two real numbers.
Assume that the action is not faithful. That is, there are !" !# so that
!" a<" <8 b !# a<" <8 b, or a!" <" !" <" b a!# <" !# <8 b
for all a<" <8 b 8 . Pick some a<" <8 b ! with say <5 ! for some " 5 8. Then comparing the
5 -th components, !" <5 !# <5 , so that if we take <"5 , then using the definition of an action,

< 5 a ! " <5 b


"
<"5 !" <5 < 5 a! # <5 b
"
5" !# <5 .

Since the elements here are computed as multiplication of two real numbers, <"5 !3 <5 !3 for 3 e" #f so
that !" !# . By definition, this means the action must be faithful. 

Problem 1.7.8 Let E be a nonempty set and let 5 be a positive integer with 5 kEk. The symmetric group
WE acts on the set F consisting of all subsets of E of cardinality 5 by 5 e+" +5 f e5a+" b 5 a+5 bf.
(a) Prove that this is a group action.
Robert Krzyzanowski Solutions to Dummit and Foote Chapter 1

(b) Describe explicitly how the elements a" #b and a" # $b act on the six #-element subsets of e" # $ %f.

Proof. (a) Let 5" 5# WE . Then for all e+" +5 f F ,


5" a5# e+" +5 fb 5" e5# a+" b 5# a+5 bf e5" a5# a+" bb 5" a5# a+5 bbf
ea5" 5# ba+" b a5" 5# ba+5 bf a5" 5# b e+" +5 f.
To show the second property of group actions, take any e+" +5 f F ; then
" WE e+" +5 f e"WE a+" b "WE a+5 bf e+" +5 f.
a" #b e" #f e# "f, a" #b e" $f e# $f a" #b e" %f e# %f
a" #b e# $f e" $f a" #b e# %f e" %f a" #b e$ %f e$ %f.
(b)

a" # $b e" #f e# $f, a" # $b e" $f e# "f a" # $b e" %f e# %f


a" # $b e# $f e$ "f a" # $b e# %f e$ %f a" # $b e$ %f e" %f.

by e5 WE 5a,b , a, F f, that is, the permutations that fix all , F. 


Notice that in general, the stabilizer (see Problem 1.7.4) of a subgroup F E acted upon by 5 WE is given

Problem 1.7.9 Do both parts of the preceding exercise with "ordered 5 -tuples" in place of "5 -element

that, for example, the #-tuple a" #b and a# "b are different even though the sets e" #f and e# "f are the
subsets," where the action on 5 -tupes is defined as above but with set braces replaced by parentheses (note

same, so the sets being acted upon are different).

Proof. (a) Let 5" 5# WE . Then for all a+" +5 b F ,

5" a5# a+" +5 bb 5" a5# a+" b 5# a+5 bb 5" a5# a+" bb 5" a5# a+5 bb

a5" 5# ba+" b a5" 5# ba+5 b a5" 5# b a+" +5 b.

To show the second property of group actions, take any a+" +5 b F ; then
" WE a+" +5 b a"WE a+" b "WE a+5 bb a+" +5 b.(b)
(b) a" #b a" #b a# "b, a" #b a# "b a" #b, a" #b a" $b a# $b a" #b a$ "b a$ #b
a" #b a" %b a# %b a" #b a% "b a% #b a" #b a# $b a" $b a" #b a$ #b a$ "b
a" #b a# %b a" %b a" #b a% #b a% "b a" #b a$ %b a$ %b a" #b a% $b a% $b
a" #b a" "b a# #b a" #b a# #b a" "b a" #b a$ $b a$ $b a" #b a% %b a% %b
a" # $b a" #b a# $b, a" # $b a# "b a$ #b, a" # $b a" $b a# "b a" # $b a$ "b a" #b
a" # $b a" %b a# %b a" # $b a% "b a% #b a" # $b a# $b a$ "b a" # $b a$ #b a" $b
a" # $b a# %b a$ %b a" # $b a% #b a% $b a" # $b a$ %b a" %b a" # $b a% $b a% "b
a" # $b a" "b a# #b a" # 3b a# #b a$ $b a" # $b a$ $b a" "b a" # $b a% %b a% %b

Notice that in general, the stabilizer (see Problem 1.7.4) of a sub-tuple F a," ,5 b E acted upon by
5 WE is given by e5 WE 5a,3 b ,3 a" 3 5f, that is, the permutations that fix all the components
of the tuple, ,3 . 

Problem 1.7.10 With reference to the preceding two exercises determine:


(a) for which values of 5 the action of W8 on 5 -element subsets is faithful, and
(b) for which values of 5 the action of W8 on ordered 5 -tuples is faithful.
Robert Krzyzanowski Solutions to Dummit and Foote Chapter 1

Proof. Call the set we will be taking subsets of E. We will make use of the following lemma:

(that is, for each W eF G f B W , we have 5aBb W ), then 5 permutes F G .


Lemma. Let E g and 5 E E be a permutation. If 5 also permutes finite subsets F E and G E

Proof. Let 5 be a permutation of E, F and G . Assume there is an B F G such that 5aBb F G , say
5aBb F without loss of generality. This contradicts the fact 5 permutes G (since that fact says 5aBb G ).
Hence, 5 permutes F G .

permutation on + E3 .
Corollary. If E g 5 E E is a permutation, and E3 E are subsets of E, then, 5 is also a

Since E is finite, index it by E e+3 f83" where 8 kEk.

(a) If 5 !, every permutation of g is g, so the action is not faithful. From Problem 1.7.6, all we need to

then 5ae+3 fb e+3 f for each " 3 8, so that 5a+3 b +3 for +3 E; then by definition 5 ". Thus, for
determine is the kernel of the action. If 5 ", then if " W 5 W W for each one-element subset W of E,

W eW E kW k 5f it is true " W 5 W W . For each " 3 8 and " 4 8 with 3 4, let


5 " the action is faithful. Let " 5 8. Assume there is a non-trivial permutation 5 on E so that for each

W34 E e+3 f so that +4 W34 with kW34 k 5. We know such a set exists for each 3 4 with 3 4, since
kW34 k 5 8 " kE e+3 fk. The fact 5 " guarantees W34 is non-empty. Then we claim that
f4 + W34 e+4 f for " 4 8.
"38
34

+< W<4 and since f4 W<4 , this is a contradiction. Thus, f4 e+4 f. Then apply the lemma to the subsets
First, notice +4 f4 since by definition +4 W34 for each W34 . Assume there is some other +< W34 . But

W34 This means 5 is a permutation on f4 e+4 f. Hence, 5ae+4 fb e+4 f and thus 5 a+4 b +4 . This holds for
each " 4 8, so that 5 ". We have shown explicitly that " is the only permutation in the kernel of the
action, so that for " 5 8 the action is faithful. Finally, if 5 8, the only 8-element subset of E is E
itself, and " E 5 E E is true for any permutation 5, since 5 E im 5 E, so that the action is not
faithful.
In conclusion, if 5 ! or 5 8 then the action is not faithful, and otherwise (" 5 8) it is faithful.
This stems from the fact there are at least 8 5 -element subsets when the latter condition holds, some of which
we can intersect and apply the lemma to.
(b) If 5 !, every permutation of the !-tuple is the !-tuple, so the action is not faithful. Otherwise, again
from Problem 1.7.6, we only need to determine whether the kernel of the action is just the identity. For

tuple a+3 +3 b where +3 is written 5 times. Assume the action is not faithful, that is, there is a permutation 5
5 !, let the 5 -tuple X3 (" 3 8) be such that each component of X5 is +3 . In other words, X5 is just the

so that " X 5 X 5 for each 5 -tuple X . In particular, this is true for each X3 , so that
5 a+3 +3 b a5a+3 b 5 a+3 bb a+3 +3 b,
and hence component-wise 5a+3 b +3 . But we can do this for each " 3 8, so 5 ". Then the kernel of
the action is just ", so the action is faithful. Therefore, the action is faithful for all ! 5 8 and not when
5 !. 

Problem 1.7.11 Write out the cycle decomposition of the eight permutations in W% corresponding to the
elements of H) given by the action of H) on the vertices of a square (where the vertices of the square are
labelled as in Section 2).
Robert Krzyzanowski Solutions to Dummit and Foote Chapter 1

Proof. Label the vertices as below.

Then the rotations < <# <$ correspond to the maps a" # $ %b a# $ % "b a" # $ %b a$ % " #b and
a" # $ %b a% " # $b, respectively. Hence, the permutations corresponding to the actions by " < <# and
<$ are " a" # $ %b a" $ba# %b, and a" % $ #b, respectively. Similarly, if we let = be the reflection with fixed

corresponding to the actions by = =< =<# and =<$ to be a" $b a" %ba# $b a# %b and a" #ba$ %b,
points in the upper right and lower left corner (# and % in the original square), we can find the permutations

respectively. 

Problem 1.7.12 Assume 8 is an even positive integer and show that H#8 acts on the set consisting of pairs of
opposite vertices of a regular 8-gon. Find the kernel of this action (label vertices as usual).

into e3 3 7f, for " 3 7. Let = be the reflection that fixes the points that " and 7 are at originally.
Proof. Label the vertices as in Section 2 and let 8 #7. Then the pairs partition the set of vertices

Assume =<5 is in the kernel for some " 5 8. But, notice that then
=<5 e" 7 "f <5 = e" 7 "f <85 e" 7 "f e8 " 5 8 " 7 5 f,

7 " unless 5 8 (i.e., =<5 =) or 5 7. In the former case, = e# 7 #f e8 " 7f so that = does
where we let 8 " 8 # #8 be the same points as " # 8. However, notice 8 " 5 is not " or

not fix all vertices either. In the latter case, =<7 e# 7 #f <7 = e# 7 #f <7 e8 " 7f
e7 " 8f so that =<7 does not fix all vertices either. Hence, no element of the form =<5 can be in the kernel
of the action.
Now consider <5 for " 5 8 with 5 7. Then <5 e" 7 "f e5 " 5 7 "f so that these
two sets are not equal unless 5 8 (so that <5 ") or 5 7. The former is the trivial case (the identity), so
consider the latter. We have <7 e3 3 7f e3 7 3 #7f e3 3 7f for each " 3 7. Hence,
<5 is in the kernel of the action. In conclusion, the kernel is e" <7 f. Notice this also means the action is not
faithful. 

Problem 1.7.13 Find the kernel of the left regular action.

Proof. Let K K K be given by 1 + 1+ as group multiplication in K (for + 1 K). Then the


kernel of this left regular action is the set of 1 K such that 1+ + for all + K. However, notice that for
any + K, 1+ + implies by right cancellation that 1 ". Thus, the kernel of the left regular action is just
the identity (so that it is a faithful action). 

Problem 1.7.14 Let K be a group and let E K. Show that if K is non-abelian then the maps defined by
1 + +1 for all 1 + K do not satisfy the axioms of a (left) group action of K on itself.

action, then if we fix + K, 1" a1# +b 1" a+1# b +1# 1" should be equal to a1" 1# b + +1" 1# . But this
Proof. If K is non-abelian, it has two non-commuting elements 1" 1# . If the given map was a group

is saying +1# 1" +1" 1# which by left cancellation implies 1# 1" 1" 1# , contradicting the fact 1" and 1# are
non-commuting elements. 

Problem 1.7.15 Let K be any group and let E K. Show that the maps defined by 1 + +1" for all
1 + K do satisfy the axioms of a (left) group action of K on itself.
Robert Krzyzanowski Solutions to Dummit and Foote Chapter 1

Proof. Let 1" 1# K. Then


1" a1# +b 1" a+1"
# b +1# 1" +a1" 1# b
" " "
a1" 1# b +,
and if we let " be the identity in K, then " + +a"" b +. 

Problem 1.7.16 Let K be any group and let E K. Show that the maps defined by 1 + 1+1" for all
1 + K do satisfy the axioms of a (left) group action (this action of G on itself is called conjugation).

Proof. Let 1" 1# K. Then


1" a1# +b 1" a1# +1"
# b 1" 1# +1# 1" a1" 1# b+a1" 1# b
" " "
a1" 1# b +,
and if we let " be the identity in K, then " + a"b+a"" b +. 

Problem 1.7.17 Let K be a group and let K act on itself by left conjugation, so each 1 K maps K to K by
B 1B1" .

of K). Deduce that B and 1B1" have the same order for all B in K and that for any subset E of K, kEk
For fixed 1 K, prove that conjugation by 1 is an isomorphism from K onto itself (i.e., is an automorphism

k1E1" k (here 1E1" e1+1" l + Ef).

Proof. Let B C K. Then 1BC1" 1B1" 1C1" a1B1" ba1C1" b so that conjugation is a

then if we conjugate B 1" C1 K we get 1B1" 1a1" C1b1" C. This shows conjugation is a
homomorphism. Now assume 1B1" 1C1" . Then right and left cancellation yield B C. Finally, if C K,

bijection, so in conclusion it is an isomorphism from K to itself. From Problem 1.6.2, we know B and 1B1"
must have the same order for all B in K. Finally, if B E, then B 1E1" by definition. Conversely, if

1C1" 1E1" ; but 1C1" 1a1" B1b1" B. Hence, conjugation gives a bijection from E to 1E1" . But
B 1E1" , then by definition B 1C1" for some C E so that C 1" B1. But since C E, by definition

from E to 1E1" . But if E z 1E1" , then kEk k1E1" k. 


since conjugation is a homomorphism on K, it must be a homomorphism on E, so that it is an isomorphism

Problem 1.7.18 Let L be a group acting on a set E. Prove that the relation on E defined by
+, if and only if + 2, for some 2 L
is an equivalence relation. (For each B E the equivalence class of B under is called the orbit of B
under the action of L . The orbits under the action of L partition the set E).

+ , E and assume + ,. Then + 2, for some 2 L . But then 2" + 2" a2,b a2" 2b, ", ,,
Proof. To show reflexivity, notice by one of the action axioms that + " +, so that + +. Now let

some 1 L . Hence, + 2, 2a1- b a21b- so that + - . 


so that , +. Finally, assume + , - E with + , and , - . Then + 2, for some 2 L and , 1- for

Examples. Since the book provides none, the following examples are given, intended to introduce a more
intuitive understanding of the notion of an orbit.

the orbit of each element B for this action is just eBf. Call this the identity orbit. Then, for example, this
(1) If L E K for K a group, then for + , E, + 2, for some 2 L if and only if 2 ". Therefore,

implies the action given in Problems 1.7.2 induces the identity orbit on each element in .
(2) The actions given in Problem 1.7.11 and 1.7.12 induce the orbit of the set itself for each element. In other
words, all elements in this action are equivalent under the relation given in the previous problem (since we
can "unrotate" and "unreflect" any rotation and reflection of the vertices). Additionally, if the group in
Robert Krzyzanowski Solutions to Dummit and Foote Chapter 1

Problem 1.7.11 was taken to be the subgroup e" < <# <$ f H) , then the rotations of the square would be in
one equivalence class (orbit), and the reflections would be in another (so that the action has two distinct
orbits).
(3) If L a b and E 8 , then the orbit of a+" +8 b E is e!a+" +8 b ! L f. Therefore,
we can say if 8 " then E z 8" (real projective space), where is the equivalence relation in the
previous problem. 

Problem 1.7.19 Let L be a subgroup of the finite group K and let L act on K (here E K) by left
multiplication. Let B K and let b be the orbit of B under the action of L . Prove that the map
L b defined by 2 2B
is a bijection (hence all orbits have cardinality kL k). From this and the preceding exercise deduce Lagrange's
Theorem:
if K is a finite group and L is a subgroup of K then kL k divides kKk.

Proof. Let 2 1 L . Then if 2B 1B, right cancellation yields 2 1. Hence the map is injective.

surjective. Therefore, it's a bijection. Since K is finite, L is finite, so that kL k kbk for each orbit b, since
Now assume that C b. By definition of orbit there is an 2 L so that C 2B, so that the map is also

there is a bijection between them. Since the orbits partition K as a set by the previous exercise,
K - bB and the bB are disjoint,
BK
where bB is the orbit of B under the action. Hence,

k K k - b B ! k b B k 8 kb k 8 k L k
BK BK

bB are disjoint. Therefore, by definition kL k divides kKk. 


where 8 is the number of distinct orbits of the action and where the second equality follows from the fact the

For the problems below, Geometry: Euclid and Beyond by Robin Hartshorne is a useful very elementary
further treatment on geometry.

Problem 1.7.20 Show that the group of rigid motions of a tetrahedron is isomorphic to a subgroup of W% .

induces a permutation a1 K given by 51 E E 51 a+b 1 +, and that these permutations in turn


Proof. Denote the group by K. Earlier in the chapter, we showed that a group K acting on a set E

induce a homomorphism : K WE given by :a1b 51 . Now label the four vertices of the tetrahedron by
" # $ and %. Then if E e" # $ %f, we have an induced homomorphism : K W% given by how each
rigid motion 1 K permutes the vertices of the tetrahedron. By Problem 1.6.13, notice that :aKb is a
subgroup of W% . Moreover, since each rigid motion gives a distinct permutation of the vertices of a

1.6.13, this means K z :aKb. 


tetrahedron (and is completely determined by where it sends the vertices), : is injective, so again by Problem

Problem 1.7.21 Show that the group of rigid motions of a cube is isomorphic to W% .

induces a permutation a1 K given by 51 E E 51 a+b 1 +, and that these permutations in turn


Proof. Denote the group by K. Earlier in the chapter, we showed that a group K acting on a set E

induce a homomorphism : K WE given by :a1b 51 . Now label the four pairs of opposite vertices of
the tetrahedron by " # $ and %. Then if E e" # $ %f, we claim K acts on E. First, obviously the identity
Robert Krzyzanowski Solutions to Dummit and Foote Chapter 1

rigid motion (not moving any points) preserves the vertices and hence the pairs, so that " 3 " for
" 3 %. Now let 1" 1# K be rigid motions Let 3 E. Then if we first apply 1" to 3 and then 1# to the

first performing 1" and then 1# ). Hence, 1# a1" 3b a1# 1" b 3.


resulting pair, 3 will be mapped to the same pair as if we had applied 1# 1" (the successive motion given by

Since K acts on E, we have an induced homomorphism : K W% (as seen earlier) given by how each
rigid motion 1 K permutes the pairs of vertices of the tetrahedron. Since there is only one rigid motion that

From Section 1.3 we know kW% k %x #%, and from Problem 1.2.10 we know kKk #%, so that kKk kW% k.
fixes all four pairs (namely, " K), by Problem 1.7.6 the action of K on E is faithful, so that : is injective.

Then : is also surjective (since the groups are finite), so K z W% . 

Problem 1.7.22 Show that the group of rigid motions of an octahedron is isomorphic to W% . Deduce that the
groups of rigid motions of a cube and octahedron are isomorphic.

induces a permutation a1 K given by 51 E E 51 a+b 1 +, and that these permutations in turn


Proof. Denote the group by K. Earlier in the chapter, we showed that a group K acting on a set E

induce a homomorphism : K WE given by :a1b 51 . Now label the four pairs of opposite faces of the
octahedron by " # $ and %. Then if E e" # $ %f, we claim K acts on E. First, obviously the identity rigid
motion (not moving any points) preserves the vertices and hence the pairs, so that " 3 " for " 3 %.
Now let 1" 1# K be rigid motions Let 3 E. Then if we first apply 1" to 3 and then 1# to the resulting pair,

1" and then 1# ). Hence, 1# a1" 3b a1# 1" b 3.


3 will be mapped to the same pair as if we had applied 1# 1" (the successive motion given by first performing

Since K acts on E, we have an induced homomorphism : K W% (as seen earlier) given by how each
rigid motion 1 K permutes the pairs of faces of the octahedron. Since there is only one rigid motion that

From Section 1.3 we know kW% k %x #%, and from Problem 1.2.11 we know kKk #%, so that kKk kW% k.
fixes all four faces (namely, " K), by Problem 1.7.6 the action of K on E is faithful, so that : is injective.

Then : is also surjective (since the groups are finite), so K z W% . By the previous exercise, the group of rigid
motions of a cube is also isomorphic to W% , and hence to the group of rigid motions of an octahedron. 

Problem 1.7.23 Explain why the action of the group of rigid motions on the set of three pairs of opposite
faces is not faithful. Find the kernel of this action.

Proof. Last problem...will finish in a bit.

The authors write "to a subgroup of W% " and not "to W% ", but see the errata: www.emba.uvm.edu/~foote/errata_3rd_edition.pdf.

Vous aimerez peut-être aussi